Lower Limb Discussion Questions

Ace your homework & exams now with Quizwiz!

a. Clubfoot

Talipes Equinovarus is known as what? a. Clubfoot b. Flat feet c. Claw toes d. Hammer toe

D) Anterior Tibial (Dorsalis Pedis)

The Talus is supplied by three arteries: Perforating Peroneal artery, Posterior Tibial and the.... A) Plantar Arch Arteries B) Medial Plantar Artery C) Dorsal Digital Arteries D) Anterior Tibial (Dorsalis Pedis)

C. Ischial tuberosity

The body's weight rest on which tuberosity when sitting? A. Iliac tuberosity B. Deltoid tuberosity C. Ischial tuberosity D. Tibial tuberosity

b.lateral

The femoral nerve sits _____to the femoral sheath? a.medial b.lateral c.distal d.proximal

c) 10-12

The great saphenous vein has _______________ valves, which are more numerous in the leg than the thigh. a) 4-6 b) 20-22 c) 10-12 d) 14-16

a) digital nerves

The intermetatarsal, metatarsophalangeal, and interphalangeal joints are innervated by: a) digital nerves b) deep fibular nerve c) sural nerve d) plantar nerve

a) femur

The longest and heaviest bone in the body is the: a) femur b) tibula c) fibula d) humerus

C . T12-L5

The psoas major muscle arises in the abdomen from the intervertebral discs, sides, and transverse process of: A . C5-T1 B . L4 C . T12-L5 D . T7-8

C. Tibial nerve and common fibular nerve

The sciatic nerve usually ends at the superior angle of the popliteal fossa by dividing into what two nerves? A. Tibial nerve and median sural cutaneous nerve B. Saphenous nerve and sural nerve C. Tibial nerve and common fibular nerve D. Tibial nerve and saphenous nerve E. Saphenous nerve and common fibular nerve

a. anterior cruciate ligament and posterior cruciate ligament

The two cruciate ligaments of the knee are: a. anterior cruciate ligament and posterior cruciate ligament b. anterior cruciate ligament and medial cruciate ligament c. posterior cruciate ligament and medial cruciate ligament d. posterior cruciate ligament and lateral cruciate ligament

a. psoas major and iliacus

The two muscles that make up the iliopsoas muscle are: a. psoas major and iliacus b. psoas minor and iliacus c. obturator externus and iliacus d. obturator internus and iliacus

FALSE

True or False: You should get a massage following a compete rupture of a hamstring.

D. Sartorius, Gracilis, Semitendinosus

What are the three muscles that make up the Pes Anserinus common tendon? A. Rectus Femoris, Gracilis, Sartorius B. Extensor digitorum, Rectus Femoris, Pectineus C. Semimembranosus, Semitendinosus, Biceps Femoris Long Head D. Sartorius, Gracilis, Semitendinosus

b. Enlarge

What happens to lymph nodes when they die? a. Shrink b. Enlarge c. Absorb d. Harden

a) Coxa Vara

What hip injury is related to a weakened epiphysial plate in the femoral neck, acute trauma/repetitive micro traumas with abduction and lateral rotation of the thigh, and dislocation (slip) of the femoral neck epiphysis? a) Coxa Vara b) Courtney Cox c) Femoral neck epiphysitis d) Hip arthralgia

c. sartorius

What is the longest muscle in the body? a. gracilis b. biceps femoris c. sartorius d. rectus femoris

A. Gracilis

Which muscle is often transplanted to replaced a damage muscle? A. Gracilis B. Sartorius C. Gemelli D. Piriformis

e. Both C & D Extensor digitorum longus & Fibularis tertius

Which muscle(s) are part of the antierior compartment of the leg? a. Gastrocnemius b. Flexor digitorum longus c. Extensor digitorum longus d. Fibularis tertius e. Both C & D

a) Ilium

Which part of the hipbone is the largest? a) Ilium b) Ischium c) Pubis d) Obturator foramen

b. continued normal bone growth may be affected

A 12-year-old boy presents to your clinic with knee pain, tenderness, and swelling at his tibial tubercle. Given the child's age and presenting complaints, you believe the boy has Osgood-Schlatter disease. You order an x-ray to confirm your suspicions and while the boy does have Osgood-Schlatter disease, he also has a fracture of his tibia. What is a concern you have regarding this boy's tibial fracture IF the epiphysial plate was involved in the fracture? a. this boy is now susceptible to tibial dislocations b. continued normal bone growth may be affected c. fibula is likely to fracture d. tibia is likely to fracture again

a) Superior gluteal nerve

A 26-year-old female presents to the clinic for a routine physical. You notice that she has a "waddling" or gluteal gait. Injury to which nerve is associated with this type of gait? a) Superior gluteal nerve b) Inferior gluteal nerve c) Superior clunial nerve d) Sciatic nerve

a) Ischial bursitis

A 28 year old professional cyclist comes in with pain localized to lateral upper thigh. Her pain increases with movement of the gluteus maximus. You suspect which disease due to the repeated stress from cycling? a) Ischial bursitis b) Sciatica c) IT band tightness d) Hamstring injury e) Broken neck of femur

b) saphenous nerve

A 30 y/o woman who is an IV drug user presents for follow up. She is s/p a cutdown surgery in her lower leg to insert a cannula for medications, since percutaneous venous access was insufficient. She now complains of pain and numbness along the medial border of the right foot. Which nerve was likely cut during this surgery? a) sciatic nerve b) saphenous nerve c) deep fibular nerve d) sural nerve

b) Femoral nerve

A 32 y/o women is in the OR needing a procedure on her lower leg. You determine the best course of action to be a regional nerve block of the lower limb instead of general anesthesia. You initiate the block 2 cm inferior to the inguinal ligament, approximately one finger breadth lateral to the femoral artery. What nerve are you blocking? a) Obturator nerve b) Femoral nerve c) Genitofemoral nerve d) Gluteal nerve

d) Femoral hernias are a protrusion of abdominal viscera, often a loop of the small intestine.

A 33-year-old female presents to the clinic after noticing a tender, small, protruding mass in her femoral triangle. After a physical exam and some labs/imaging, you come to the conclusion that she has a femoral hernia. Which piece of information would be appropriate/correct to include while educating her on her diagnosis? a) Femoral hernias are not common in females, so you are very concerned. b) Multiple pregnancies decrease her risk for femoral hernias. c) Her femoral hernia will remain the presenting size and will not cause any complications. d) Femoral hernias are a protrusion of abdominal viscera, often a loop of the small intestine.

b) Babinski sign; no it is not normal for his age

A 7-year-old boy is in the clinic today for a neurologic examination. While testing his plantar reflex by stroking the lateral aspect of the sole of the foot with a blunt object, his lateral four toes fanned out and the great toe dorsiflexed. What is this response called and is it normal for a child his age? a) Babinski sign; yes it is normal for his age b) Babinski sign; no it is not normal for his age c) Pronator sign; yes it is normal for his age d) Pronator sign; no it is not normal for this age

b. Palpitations

A 88-year-old male presents to the clinic with right foot pain. The PA is suspicious for acute arterial occlusion. Which exam finding would not be consistent with this? a. Pain b. Palpitations c. Pallor d. Paresthesia e. Paralysis f. Pulselessness

f. C & D (Superiorly and inferiorly)

A patient presents with severe pain on the posterior side of her knee. You palpate and determine the cause of the pain is a popliteal abscess. You are concerned about it spreading, in which direction(s) would it spread? a. Medially b. Laterally c. Superiorly d. Inferiorly e. A and B f. C and D

b. Automobile accidents

Acquired dislocation of the hip joint is uncommon due to the strong and stable joint, but when does this most often occur? a. Walking b. Automobile accidents c. Slipping and falling d. Hockey

B. Fasciotomy of overlying fascia

Adam Theilen comes into your office complaining that he got hit very hard on his right thigh during the football game on Sunday. As you perform your peripheral vascular exam, you cannot palpate a pulse in his right dorsal pedis artery and become concerned about compartment syndrome. What is the treatment for compartment syndrome? A. Excision of the dorsal pedis artery B. Fasciotomy of overlying fascia C. Phlebectomy D. Light stretching and pool therapy

a) paresthesia in the foot

After an anesthetic injection a few cm inferior to the midpoint of the PSIS and the superior border of the greater trochanter, what is a common symptom? a) paresthesia in the foot b) paresthesia in the hand c) pan cytopenia d) loss of shoe sign

a) Gracilis

After an unfortunate accident in the weight room, Tiny Tanner finds himself unable to eat chicken nuggets due to a loss of function in his hand. He comes to your clinic several months later wondering if there is any chance he will ever be able to eat his nugs again. You inform him that there is a muscle in his leg that can be removed without a noticeable loss and replaced into his forearm and hand to regain function. What muscle would you remove from the thigh to restore digital flexion in his hand? a) Gracilis b) Sartorius c) semitendinosus d) adductor longus

b. Tibial Nerve

All of the muscles of the posterior compartment of the leg are innervated by which nerve? a. Femoral Nerve b. Tibial Nerve c. Deep Fibular Nerve d. Superficial Fibular Nerve e. Lumbar Nerve

A: Ischial bursitis from recurrent microtrauma caused by her cycling

An avid cyclist presents to your clinic as she has been finding sitting has been painful recently. On exam you find that she has pain to palpation over her ischial tuberosity and increased pain with use of her gluteus maximus. She denies any recent trauma. What diagnosis is most likely? A: Ischial bursitis from recurrent microtrauma caused by her cycling B: Ischial tuberosity stress fracture caused by her cycling C: Muscle strain from cycling too much D: Greater trochanter stress fracture from cycling

c. age 10-12

At what age does the patella completely ossify? a. age 2 b. age 25-30 c. age 10-12 d. age 3-4

C. Hyaline cartilage

Bart Simpson comes to see you in clinic because of the musculoskeletal pain he has been having in his leg. You notice from his history and health records that Bart has been going through a growth spurt recently. His Mother, Marge complains of having to buy Bart new pants halfway through the school year because of his growth and his persistence of wearing out the knees while playing soccer. You attribute Bart's pain to irritation of his epiphysial plates which are made out of what kind of cartilage? A. Fibrocartilage B. Elastic Cartilage C. Hyaline cartilage D. Dense irregular cartilage

C. Apply heat

Bill was skiing uphill when he felt something snap in his hamstring and subsequently a large contusion developed on his thigh. Which of the following is not a recommended treatment for a thigh contusion and may cause further injury? A. Apply a bandage to protect and support injury B. Elevate the leg C. Apply heat D. Apply compression wrap E. Apply cold therapy

c) Tibia and Fibula

Bryce is going skiing in the mountains for the first time on a vacation get away. He is a very inexperienced skier and had a bad fall where his ski did not release. He is in a lot of pain and is suspected of having a boot top fracture. What bone(s) are broken in this type of fracture? a) Tibia b) Fibula c) Tibia and Fibula d) Femur

d) Ischial Bursitis

Dainty Derek decided to have a friendly competition over Thanksgiving where he totally cheated and gained 11lbs. His wife asked him politely to shed the newly gained weight, to where he reluctantly agreed. He comes into the clinic about a month after starting a rigorous cycling workout program, complaining of hip pain radiating to his buttocks. Understanding that repetitive movements of hip extensions and microtrauma in the hip region may cause what? a) Trochanteric Bursitis b) Sciatic Nerve Injury c) Hamstring micro tear d) Ischial Bursitis

a) tarsal tunnel syndrome

Dan is experiencing heel pain and decides to get it checked out. The doctor notes that he has edema and tightness in the ankle, specifically from the medial malleolus to the calcaneus. He suspects the flexor retinaculum in the posterior compartment of the leg is involved. Given the symptoms, the location, and suspected involvement of the flexor retinaculum, it is likely the tibial nerve is being compressed. What is the syndrome called? a) tarsal tunnel syndrome b) flexor retinaculum syndrome c) plantar fascia syndrom d) medial malleolus syndrome

c. Drawer Test

Danner and Tereck were playing some pick-up football with Jam and Tomes. Tereck was showing off his "sweet moves" and dodged one of Tomes's tackles by suddenly stopping, causing his femur to slide forward on top of his fibula. Tereck heared a "pop" and his knee immediately gave way. What test can you perform to see if the ACL and/or PCL were torn and the degree of the injury? a. Extension of leg against resistance b. Flexion of leg against resistance c. Drawer Test d. Calcaneal Reflex test

d) gracilis strain

Derek and Tanner decided to race in the parking lot after class after a heated debate over the gracilis muscle. Derek started off in a dead sprint and felt immediate pain of his anteromedial thigh muscles. What injury did Derek most likely experience? a) groin strain b) femur fracture c) tibial fracture d) gracilis strain

B. Great saphenous vein

Emily is in cardiogenic shock and is need of IV inotrope. Her veins are collapse, the cardiac PA taking care of Emily did an emergency procedure call "saphenous cutdown". Which vein did the PA most likely insert the canula to administer the IV inotrope? A. Femoral vein B. Great saphenous vein C. Lesser saphenous vein D. Iliac vein

TEST BOTH: evert foot strongly against resistance. Muscle tendons can be seen/palpated INFERIOR to the lateral malleolus * When you stand on one foot, the Fibularis Longus helps to steady the leg on the foot

Fibularis longus and brevis The Evertors How do you test the superficial fibular nerve?

d. 85%

Foot ulcers precede what percentage of diabetes related amputations? a. 65% b. 70% c. 5% d. 85%

a. Plantar fasciitis

For the past week, Tanner has been complaining of pain on the bottom of his foot and heel. He has not told anyone that he teaches a high-impact pilates class Monday-Friday, so he does not know how to describe the reason he is having this pain. After using your interviewing skills gained in H&P, he finally admits what he does every day after class and you can diagnose what is happening to him. What is your diagnosis? a. Plantar fasciitis b. fractured patella c. shin splints d. Osgood-Schlatter's disease

D. use a sharp object and ask if pain is felt

How do you test for pain sensation? A. use a soft object and ask if pain is felt B. ask the patient if they feel pain at base line C. use a cold object and ask if pain is felt D. use a sharp object and ask if pain is felt

d. 2 muscles

How many muscles are considered dorsal muscles of the foot? a. 4 muscles b. 8 muscles c. 14 muscles d. 2 muscles

A: Sartorius

If a patient is having difficulty laterally rotating his thigh against resistance what muscle is most likely not working? A: Sartorius B: Rectus femoris C: Pectineus D: Psoas minor

d. All of the above Trunk inferior to Umbilicus, Perineum, Entire Lower Limb

If upon physical examination of a patient you note palpable enlarged inguinal lymph nodes, where should you examine to determine their cause of enlargement? a. Trunk inferior to Umbilicus b. Perineum c. Entire Lower Limb d. All of the above

b) common fibular nerve

Injury to what nerve can cause "foot drop"? a) tibial nerve b) common fibular nerve c) deep fibular nerve d) superficial fibular nerve

c) sesamoid

Jane is an avid long distance runner. Her goal over the past year has been to run one marathon per month. A week after running Grandma's Marathon in Duluth, she notices her right foot is slightly swollen, and her big toe is tender to touch and has limited range of motion. She goes in to urgent care and the doctor orders an x-ray of her foot. The x-ray shows that theres a fracture in her big toe at the base of where the tendon of the flexor hallucis longus would likely be. Which bone in her big toe is fractured? a) metatarsal b) distal phalange c) sesamoid d) proximal phalange

c. Tibialis anterior strain

Jenny, 23-year-old female, presents to the clinic for swelling and pain within her lower legs. A thorough HPI was completed by the first-year PA student. Social history was significant for training for an upcoming marathon. Jenny explains she just "hops on the treadmill and start running". She frequently skips walking and stretching after running this 15-20 miles at a time. What condition is Jenny most likely experiencing? a. Calcaneal tendinitis b. Gastrocnemius strain c. Tibialis anterior strain d. Plantar fasciitis

b) Krazy Kelsey, DVT

Jinormous James was on a long plane ride with his good friend Krazy Kelsey. JJ complained he was too tall for the seats, so often on the plane ride he got up and walked about the cabin (ignoring the seat belt lights and any apparent danger because he does not believe in fear). Krazy Kelsey on the other hand, sat content in her seat with no interruptions while drinking a sangria (sangrias?). After the flight, ONE of the two had swelling and pain in their lower right leg. There were no symptoms in the left leg. Prior to the flight (19 days ago), Jinormous James had a left index finger nail matrix ablation. Which of the two plane riders would most likely be affected, and what is the most probable diagnosis? a) Jinormous James, DVT b) Krazy Kelsey, DVT c) Krazy Kelsey, peripheral vasculitis d) Jinormous James, peripheral vasculitis

c. Transverse march fracture

John Doe has been inactive for several years. However, today he became motivated and decided to go for a long hike in Chester Park. Unfortunately, during his adventure he experienced severe pain in his right leg to the point that he was no longer able to walk. He called 911. Upon arrival to the ED, a radiograph was obtained, which showed a fracture of the inferior third of his tibia. What type of fracture is this? a. Oblique fracture b. Greenstick fracture c. Transverse march fracture d. Comminuted fracture

a) Tibial Nerve

John is a 17 year old male who presents to your office after an ankle sprain which caused edema and tightness in the ankle (involving the synovial sheaths). Even though his ankle has since gotten better, he is now having pain in his heel with continued ankle swelling. What nerve did the swelling likely compress? a) Tibial Nerve b) Fibular Nerve c) Saphenous Nerve d) Femoral Nerve

d) trochanteric bursitis

Kimberly has been working really hard to get in shape for spring break. She tells the PA that she has been taking the stairs instead of the elevator at work and most of her workouts include steeply elevated treadmill training. While talking about the pain, Kimberly touches her gluteus medius area. What is most likely the cause of Kimberly's pain? a) hamstring injury b) injury to superior gluteal nerve c) injury to sciatic nerve d) trochanteric bursitis

B . The valves in your veins are incompetent due to dilation or rotation and let blood back through, causing pooling

Lauren, 45 year old female, presents to the clinic today with pain and discomfort in her left lower limb. You identify varicose veins and prescribe compression socks for 3 months. How do you explain the mechanism of the disease to the patient? A . The valves in your veins are locked shut and don't let any blood through B . The valves in your veins are incompetent due to dilation or rotation and let blood back through, causing pooling C . Your arteries are clotted extensively and blood cannot get to your lower legs D . Your veins are clotted extensively and you are at high risk of a pulmonary embolism

a. fibularis brevis tendon

Lindsey is a 26-year-old female who presents today with right foot pain and edema. She states that the pain started during tennis practice after she "twisted her ankle". Along with a severe ankle sprain, you also diagnosed Lindsey with an avulsion fracture of the tuberosity of the 5th metatarsal. Which tendon is associated with this diagnosis? a. fibularis brevis tendon b. fibularis longus tendon c. flexor digiti minimi brevis tendon d. tibialis anterior tendon

1.Extensor digitorum longus muscle 2.Tibialis Anterior Muscle 3.Tibia 4.Flexor Digitorum Longus Muscle 5.Tibialis Posterior Muscle 6.Flexor Hallucis Longus Muscle 7.Calcaneal Tendon (Achilles Tendon) 8.Soleus Muscle 9.Peroneus Longus Tendon 10.Peroneus Brevis muscle

Lower leg (distal leg)

1.Extensor Digitorum 2.Tibialis Anterior 3.Tibialis Posterior Muscle 4.Tibial Nerve 5.Great Saphenous Vein 6.Soleus Muscle 7.Gastrocnemius Medial Head 8.Short Saphenous Vein 9.Posterior Tibial Artery Fibula

Middle Leg

a. Blood from ruptured vessels contained within fascia lata

Mike, 21-year-old male, presents to the clinic after sustaining a hamstring injury during the season's last soccer game. The injury occurred while he kicked the soccer ball with greater force, which allowed him to score the last goal! He presents to the clinic with a large hematoma as well as pain along the posterior aspect of his right leg. Mike has likely sustained a hamstring injury, what is the cause of the large hematoma? a. Blood from ruptured vessels contained within fascia lata b. Deep long bone contusion (i.e. from the femur) c. Deep muscular tissue injury resulting in discoloration over the biceps femoris d. None of these seen like a plausible explanation

A. Fracture of the sesamoid bones

Ross injured his big toe during a football game. The pain was so unbearable that he refused to walk. Which bone did Ross most likely fracture? A. Fracture of the sesamoid bones B. Fracture of metacarpal C. Fracture of femoral D. Fracture of tibial

B. Hamstring muscles

Semitendinosus, semimembranosus,and the long head of biceps femoris are refered to as what group of muscles? A. Rotator cuff muscles B. Hamstring muscles C. Knee extensor muscles D. Foot flexor muscles

a) Talus

The Tibia mainly articulates with what bone distally? a) Talus b) Calcaneus c) Navicular d) Cuneiform

a. Coxa vera, shortened

The angle of inclination between the long axis of the femoral neck and femoral shaft may change due to pathology. What term is used to describe a decreased angle? How may the length of the lower limb be affected? a. Coxa vera, shortened b. Coxa vera, lengthened c. Coxa valga, shortened d. Coxa valga, lengthened

d. adductor longus

A 35-year-old women comes into your clinic presenting with pain in the medial aspect of her left thigh. Upon questioning, you learn that the woman owns a horse farm and is often the one who is in charge of riding the horses for exercise. You perform a detailed examination and discover the woman has what is often wrongly called "riders" bones." What median thigh muscle has likely been injured? a. gracilis b. adductor magnus c. sartorius d. adductor longus

b) proximal

Desmologist Deviney was walking to class one morning following a snow fall when she slipped on the ice. In an attempt to break her fall, she contracted her quadriceps extremely quickly and hard. This caused a fracture of the patella. Which fragment of the patella is pulled off with the quadriceps tendon? a) distal b) proximal c) medial d) lateral

c) Coxa vara (Decreased angle of inclination)

Jumping James was jumping on the jumbo trampoline and jumped high off the trampoline onto the ground and jarred his leg. James has been feeling some pain in this hip that radiates to his knee joint. He decides to jump into the Jeep and go to the clinic to get it checked out. You suspect that he may have a dislocated epiphysis of the femoral head. You justify taking an X-ray of the superior end of the femoral joint. What would you expect to see on the x-ray of the joint? a) Coxa vara (Increased angle of inclination) b) Coxa valga (Decreased angle of inclination) c) Coxa vara (Decreased angle of inclination) d) Coxa valga (Increased angle of inclination)

Hop on one leg test sever pain with hopping = stress fx

Shin splints What is a test used to determine if there is stress fractures?

b. Ball and Socket

The hip is which type of multiaxial synovial joint? a. Saddle b. Ball and Socket c. Pivot d. Hinge e. Planar

c) Muscle Attachment

The main function of the Fibula is to: a) Bear weight b) Protection for the Tibia c) Muscle Attachment d) there is no function

b) neck of the femur

The most common place the femur is fractured is the a) shaft of the femur b) neck of the femur c) greater trochanter d) lesser trochanter

C. Tibial nerve

The popliteal artery lies deep to which nerve that could be compressed by an aneurysm of the popliteal artery, causing referred pain to the skin overlying the medial aspect of the calf, ankle, or foot? A. Common fibular nerve B. Sciatic nerve C. Tibial nerve D. Saphenous nerve E. Sural nerve

B. Superficial fibular nerve

Tipsy Tom is very unstable on his feet throughout the day due to his extra curricular activities. As a result, he experiences many ankle sprains while walking. He notices numbness and paresthesia at rest, and this increases with activity. What nerve is most commonly stretched to cause these symptoms? A. Deep tibial nerve B. Superficial fibular nerve C. Radial nerve D. Anterior trunk of the brachial plexus

a) sacro-iliac ligaments

Weight is transferred from the axial skeleton to the ilia, then to the femurs during standing, and to the ischial tuberosities during sitting, via what ligament(s)? a) sacro-iliac ligaments b) superior pubic ligament c) posterior sacrococcygeal ligaments d) iliolumbar ligaments

c. lateral sartorius, superior inguinal ligament, medial adductor longus

What are the borders of the femoral triangle? a. gracilis, adductor longus, biceps femoris b. gracilis, adductor magnus, biceps femoris c. lateral sartorius, superior inguinal ligament, medial adductor longus d. lateral sartorius, gracilis, pes anserinus

B. Neck

What area of the femur is most commonly fractured? A. Head B. Neck C. Middle Shaft D. Epicondyles

b. femoral

What artery is the popliteal artery a continuation of? a. posterior tibial b. femoral c. anterior tibial d. dorsal pedis

B. Tibia

What bone articulates with the condyles of the femur superiorly and the talus bone inferiorly? A. Ulna B. Tibia C. Fibula D. Calcaneous

d. Femur

What bone is the longest and heaviest bone in the body? a. Humerus b. Sacrum c. Tibia d. Femur

D. Pelvic inlet

What circular opening between the abdominal cavity and the pelvic cavity is formed by the promontory, the margin of the ala and SI joint, and the linea terminalis? A. Obturator foramen B. Greater pelvis C. Lesser pelvis D. Pelvic inlet E. Pelvic outlet

b) TB and Crohn disease

What disease can a psoas abcess result from? a) STEMI and NSTEMI b) TB and Crohn disease c) Cushing disease d) Not a disease, but rather blunt trauma

d) Make a surgical incision and drain it

What do you do if a patient presents with a well-established foot infection? a) Give them antibiotics and closely monitor b) Nothing, it should heal itself c) Soak in warm water with vinegar once daily for 15 minutes d) Make a surgical incision and drain it

d) All of the above

What evolved features are unique to the human foot? a) Medial migration of the distal attachment of the fibularis longus across the sole of the foot. b) Development of a fibularis tertius that is attached to the base of the 5th metatarsal c) Relatively everted (pronated) feet. (Causes sole to lie more fully on the ground) d) All of the above

B . the neck

What is the most commonly fractured part of the femur? A . the shaft B . the neck C . the foot D . the head

b.Quadriceps Femoris

What is the most important stabilizer of the knee? a.Rectus Femoris b.Quadriceps Femoris c.Vastus lateralis d.Adductor longus

c) quadriceps femoris

What is the most important stabilizer of the knee? a) patella b) femur c) quadriceps femoris d) tibia e) gastrocnemius

C) Fascia Lata

What is the name of the deep fascia of the thigh? A) Transversalis Fascia B) Deep fascia of leg C) Fascia Lata D) Cribriform fascia

A. Ischemia

What is the source of pain in compartment syndrome? A. Ischemia B. Lactic acid C. Bone spurs D. Fracture

A: The tibial tuberosity

What structure causes pain in Osgood-Schlatter disease? A: The tibial tuberosity B: The anterior border of the tibia C: The patella D: The head of the fibula

a. Quadriceps tendon

What tendon attaches to the superior margin of the patella? a. Quadriceps tendon b. Patellar ligament c. Biceps tendon d. IT band

TEST: dorsiflex foot (lateral 4 toes) against resistance

What test do you do to test the Deep fibular nerve?

c. complete discoid meniscus

What type of discoid meniscus fully covers the tibia? a. normal discoid meniscus b. incomplete discoid meniscus c. complete discoid meniscus d. hypermobile discoid meniscus

C) greater saphenous

What vein has high success rate for ultrasound guided IV placement in infants under inpatient care? A) lesser saphenous B) femoral C) greater saphenous D) basilic

b) Great saphenous vein

What vein is often used for coronary arterial bypass? a) Femoral vein b) Great saphenous vein c) Popliteal vein d) Profunda femoris vein

B. Dorsiflexion

When activated, the anterior compartment of the leg contains muscles that achieve what primary motion of the foot? A. Flexion B. Dorsiflexion C. Supination D. Rotation E. None of the above

B) Between 20-25 years old

When is the fusion of the three primary pelvic bones complete? A) By birth B) Between 20-25 years old C) During puberty D) Never fully fuse

c. Tibia

Which bone contains "Gerdy's Tubercle" where the IT band attaches? a. Femur b. Patella c. Tibia d. Fibula

A. Tibia

Which bone is known as the SECOND largest bone of the body? A. Tibia B. Fibula C. Humerus D. Femur

C.Fibula

Which bone is not involved in the knee joint? A. Tibia B. Femur C.Fibula D. Patella

c. Fibula

Which bone is not involved in the knee joint? a. Femur b. Tibia c. Fibula d. Patella

a) Tibia

Which bone(s) in the leg plays a role in weight-bearing? a) Tibia b) Fibula c) Tibia and Fibula d) None of the above

a) gracilis

Which muscle is the most superficial of the adductor group and also the weakest member? a) gracilis b) obturator externus c) adductor magnus d) adductor hiatus e)adductor longus

a) saphenous

Which nerve does NOT have the sciatic nerve as a true origin? a) saphenous B) tibial C) common fibular D) Sural

D . leg press (full knee extension)

Which of these is NOT a treatment for "Fat Pad Syndrome"? A . ice B . NSAIDS C . strengthening exercises D . leg press (full knee extension)

B. lateral and medial alar folds

Which structures cover the inner surface of fat pads that occupy the space on each side of the patellar ligament internal to the fibrous layer in the knee? A infrapatellar synovial fold B lateral and medial alar folds C pes anserinus D suprapatellar bursa

C) Cuboid

Which tarsus bone is cubital in shape and is the most lateral bone in distal row of tarsus? A) Navicular B) Intermediate cuneiform C) Cuboid D) Talus

b) greater saphenous vein

Which vein is formed from the union of the dorsal vein of the great toe and the dorsal venous arch of the foot? a) lesser saphenous vein b) greater saphenous vein c) obturator vein d) posterior tibial vein e) femoral vein

D: Avascular necrosis of the head of her femur

You are treating a 6 year old for a fracture of her superior femoral epiphysis. What condition are you worried about her developing in the future secondary to this injury? A: Foot drop B: Osgood-Schlatter disease C: Patellofemoral syndrome D: Avascular necrosis of the head of her femur

a. paralyzed quadriceps muscles

You have a patient come in and is walking with a forward lean and must support themselves walking by leaning on their right distal thigh with their palm with each step . What are you suspicious of? a. paralyzed quadriceps muscles b. quadriceps imbalance c. groin pull d. avulsion fracture

b. soleus

A 10-year-old boy presents to your clinic complaining of pain and edema during long periods of exercise. After taking a more extensive history and performing a thorough physical exam, you determine the location of the pain is medial to the calcaneal tendon and involves an accessory muscle. What muscle is the likely accessory muscle responsible for the boy's pain and edema? a. gastrocnemius b. soleus c. tibialis anterior d. flexor digitorum longus

d) Apophyses

A 12 yo female named Sally comes in to the ER complaining of severe hip pain. She plays soccer, basketball, is on the track team as a hurdler and competes in martial arts in her spare time working toward her black belt. Upon Xray you see she has avlused her hip bone, where do these fractures most commonly occur? a) Cartilage b) Tendon c) Bone d) Apophyses

d. Os trigonum

A 13 year old girl presents to the clinic with pain in both of her feet. She appears very muscular and from your thorough HPI you discover she has been training as a gymnast since the age of 6. She very proudly announces she trains over 5 hours a day and has been as long as she can remember. She says the pain has come on gradually but hurts the worst when she performs her floor routine where she jumps from the air and then lands in a kneeled position and sits completely on her feet. She doesn't recall twisting either of her ankles and it only hurts when she points her toes (which she does a lot in gymnastics). What developmental disformity would you expect to see on her x-ray? a. Osgood-Schlatter disease b. Coxa valga c. Coxa vara d. Os trigonum e. Calcaneal fracture

b) Popliteal: loss of leg and foot

A 13 year old girl was doing some monkey bars and they were a lot higher than she was expecting. When she fell something snapped in her leg- upon x-ray it turns out she has a distal femur fracture. What artery are we worried may have ruptured due to extensive hemorrhaging, and what could happen if this artery rupture is left untreated? a) Femoral: loss of leg and foot b) Popliteal: loss of leg and foot c) Patellar: compartment syndrome d) Tibial: compartment syndrome

d.) os trigonum syndrome

A 15 year old ballet dancer presents with bilateral swelling and pain behind her ankles, especially when "en pointe" position (plantarflexion). She says that she is worried tha she might not be able to perform at the Christmas dance recital, which she has been practicing heavily for. You order an x-ray and it reveals an accessory ossicle on the posterior aspect of both ankle joints. What is this know as? a.) osteochondritis dissecans b.) achilles tendon bursitis c.) tarsal tunnel syndrome d.) os trigonum syndrome

D Os Trigonum

A 17 year old soccer player was undergoing an x-ray of his foot after complaining of joint pain. In the x-ray a fracture was found where the secondary ossification center that becomes the lateral tuberosity of the talus fails to unite with the rest of the talus, possibly due to the forceful plantarflection from his sport. This progressed to a fracture that resulted in non-union. What is the condition? A fracture of the tibia B fracture of the sesamoid bone C Calcaneal fracture D Os Trigonum

c. extended perforating fibular artery replaced dorsal pedis artery

A 23-year-old woman presents to your clinic for her annual physical exam. You perform a peripheral vascular exam as part of this exam and cannot find a dorsal pedis pulse in either foot. What normal congenital variationcould be occurring that would explain the lack of a dorsal pedis pulse? a. acute arterial occlusion b. medial plantar nerve entrapment c. extended perforating fibular artery replaced dorsal pedis artery d. infection of foot

a) loss of dorsiflexion of the ankle causing footdrop.

A 25 y/o male hockey player presents to the ED with excruciating leg pain after he sustained an injury during a game. XR shows a right fibular neck fracture. You tell your preceptor you are worried the common fibular nerve could be injured as well, as is common with this type of fracture. Which of the following conditions would likely be caused if this nerve was severed? a) loss of dorsiflexion of the ankle causing footdrop. b) muscle injury and edema in the anterior compartment. c) inability to plantarflex ankle or flex toes. d) shin splints

b) subtalar (talocalcaneal) joint

A 25-year-old male presents to the ED after jumping off a roof. He states that his friends dared him to and that he landed primarily on his right heel. He's having trouble walking/putting weight on his foot. The xray shows a comminuted fracture of the calcaneus. This type of injury can be disabling because it disrupts which joint? a) calcaneo-cuboid joint b) subtalar (talocalcaneal) joint c) tarsometatarsal joint d) metatarsophalangeal joint

A) Calcaneal tendinitis

A 26 year old male presents to the clinic with pain in the posterior aspect of the distal right leg near the ankle while walking. The patient states that he began running intensely every day after noticing he has not been very active since beginning grad school three months ago. What is a possible source of the pain in this location? A) Calcaneal tendinitis B) Fabella sesamoid bone C) Popliteal Abscess D) Tibialis Anterior Strain (Shin splints)

C. measure the compartment pressure

A 27-year-old female was in a biking accident two days ago in which she sustained lower leg lacerations and contusions. She presents today with excruciating pain, pulselessness, pallor, and paresthesia of the same leg. What should the provider do next? A. send her home to ice it B. X-ray C. measure the compartment pressure D. put a boot on it

b) Greater saphenous vein

A 27-year-old male presents to the ED after a traumatic car accident, where he had lost a lot of blood. Your preceptor wants you to quickly make a skin incision anterior to the medial malleolus and insert a cannula for prolonged administration of blood and electrolytes. Which vein would you be inserting the cannula in? a) Lesser saphenous vein b) Greater saphenous vein c) Popliteal vein d) Tibial vein

c. fibular nerve

A 35 year old male presents to the ED after sustaining a laceration to the lateral calf 2 days ago. Inflammation and pain has spread distally and puss formation has moved proximally into the lateral popliteal fossa. Which structure does this infection course with? a. tibial nerve b. tibial artery c. fibular nerve d. plantar nerve

b. Clubfoot; shortness and tightness of features on the medial and posterior side of the foot and ankle.

A 4 month old female presents to the clinic with a foot deformity. You note that the foot is inverted, the ankle is plantarflexed, and the forefoot is adducted. What is the most likely diagnosis and main abnormality associated with this diagnosis? a. Clubfoot; shortness and tightness of features on the lateral and anterior side of foot and ankle. b. Clubfoot; shortness and tightness of features on the medial and posterior side of the foot and ankle. c. Flat foot; shortness and tightness of features on the lateral and anterior side of foot and ankle. d. Flat foot; shortness and tightness of features on the medial and posterior side of the foot and ankle.

A) Severe dorsiflexion of the foot while pressing on break

A 40 y/o man presents to the ED with extreme ankle pain after getting in a head-on collision on the way to work. The patient swerved into the opposite lane while checking his cellphone when he struck the oncoming vehicle. Xrays reveal he broke the neck of the talus bone. What likely caused this? A) Severe dorsiflexion of the foot while pressing on break B) Severe extension of the toes C) Direct blow from debris D) Heel of foot striking floor of car on impact

b) superior gluteal nerve

A 45 y/o male presents to the ED after a snowmobile accident, which dislocated his right hip. After resetting the joint, you continue to examine the patient. After asking him to stand on one leg, the pelvis on the unsupported side descends, indicating a positive Trendelenburg test. Given the history and this finding, which structure in the leg was likely damaged during the accident? a) femoral nerve b) superior gluteal nerve c) sciatic nerve d) posterior cutaneous nerve of thigh

c) softening of the articular cartilage of the patella aka "runner's knee" common in marathon runners. Can be due to overstressing of the knee region, quadriceps imbalance, or as a result from a blow to the patella or extreme knee flexion.

A 45 y/o woman presents to your clinic complaining of sore/achy knee pain, specifically around the patella. She tells you she is training for Grandma's Marathon, and has ran 1-2 marathons per year since she was 20. With this history, Chondromalacia Patellae is high on your differential. What is the best description of this injury? a) fracture of the patella b) hardening of the articular cartilage of the patella c) softening of the articular cartilage of the patella d) degeneration of the articular cartilage of patella

c) excessive and repeated friction between the skin and the patella Prepatellar bursitis is caused by excessive and repeated friction between the skin and the patella, for example, jobs associated with kneeling. The bursa may also be injured by compressive forces resulting from a direct blow or from falling on the flexed knee. If the inflammation is chronic, the bursa becomes distended with fluid and forms a swelling anterior to the knee.

A 45 y/o woman presents to your clinic with right knee pain and mild swelling anterior to the knee. Upon further questioning, she tells you she works as a preschool teacher and is often kneeling throughout the day with her students. After examination, you determine she likely has prepatellar bursitits of the right knee. Which of the following best describes the mechanism of this bursitis? a) excessive friction between the skin and the tibial tuberosity b) overuse and subsequent friction between the patellar tendon and the infrapatellar fat pad and tibia c) excessive and repeated friction between the skin and the patella d) bacteria entering the suprapatellar bursa from the torn skin

d. gracilis

A 46-year-old female has been suffering for some time from damaged flexor muscles in her right hand, particularly the flexor digitorum superficialis. After attempting numerous medications and referrals to PT, you, as the primary care provider, send the woman to a surgery consult. The surgery team comes to the conclusion that the best option for this woman is transplantation of an adductor muscle of her right leg into her right hand to replace the damaged muscles. What adductor muscle is commonly used for transplantation? a. adductor magnus b. adductor longus c. adductor brevis d. gracilis

B) Inguinal

A 50 year old woman was recently diagnosed with uterine cancer. You want to assess if it has metastasized due to the possibility of lymphatic drainage traveling from the uterine fundus. Which lymph nodes would likely be affected if metastasis had occurred? A) Axial B) Inguinal C) Popliteal D) Lumbar

e. SUCK IT UP BUTTERCUP! NO PAINS, NO GAINS

A 52 year old man presents to your clinic with pain in his anterior shins. He recently went through a traumatic experience and decided to "turn his life around." He started to eat healthier, get more sleep, and work out in a cross-fit gym. His trainers, Tiny Tanner and Doofy Derek, are big fans of box jumps and sprints. So far, he has been able to keep up and it wasn't until recently when he gets these pains in his shins that he has to stop, especially with the jumping exercises. All of the following are recommendations you can make to help alleviate the pains EXCEPT: a. Rest b. Massage c. Ice d. Stretching e. SUCK IT UP BUTTERCUP! NO PAINS, NO GAINS

d. Calcaneal bursitis

A 55 year old woman presents to the clinic with pain posterior to her right heel. She says that she is used to blisters on her heels from her marathon training, but this one feels like its deeper than the skin and is hoping you can drain it so she can get back to running. You explain to her that it isn't a blister this time. What is it? a. Shin Splint b. Fibular Nerve entrapment c. Posterior tibial artery hemorrhage d. Calcaneal bursitis e. Ruptured Calcaneal tendon

c. compression socks

A 57-year-old woman presents to your clinic for her annual physical. On exam, you note the woman has poor blood flow to her lower extremities and order an ultrasound to determine what level the blood flow is at as well as the venous return of blood back to the heart. What non-pharmacological intervention could you as the provider give to this woman to improve her venous return? a. elevate legs b. apply heat c. compression socks d. ice legs

b) calcaneal fracture, comminuted fracture

A 58 y/o male presents to the ED with severe pain in his "heel" as he describes it. He fell from a ladder while putting up Christmas lights over the weekend. What bone do you suspect is broken, and what type of fracture is it likely? a) cuboid fracture, compound fracture b) calcaneal fracture, comminuted fracture c) talus fracture, comminuted fracture d) calcaneal fracture, compound fracture

d. it has many valves

A 59 year old male presents with a STEMI, full occlusion of the LAD. He needs a CABG. Which of the following is NOT a reason to use the great saphenous vein for coronary artery bypass? a. it is readily accessible b. it is a sufficient distance between tributaries and perforating veins c. it contains a higher percentage of muscular and elastic fibers d. it has many valves

B. Popliteal aneurysm

A 60-year-old male with a history of atherosclerosis presents to the ER with a painful, swollen mass in the popliteal fossa of his right leg. When you palpate the mass you feel vibratory sensations, and when you auscultate the mass you hear bruits. Based on these two findings, what is the most likely diagnosis? A. Popliteal abscess B. Popliteal aneurysm C. Popliteal vein thrombosis D. Popliteal soft tissue sarcoma E. Popliteal lipoma

a. weakness of the quadriceps can lead to joint instability

A 65-year-old male with a history of osteoarthritis of the right knee is concerned about his "unstable" knee and moveable patella. How could you educate your patient regarding osteoarthritis? a. weakness of the quadriceps can lead to joint instability b. weakness of the hamstrings leads to joint instability c. calcification of the muscle causes loose tendons d. osteoarthritis is not related to joint instability

C: Deep vein thrombosis

A 70 year old woman presents to you with her right lower leg swollen, red, and painful. She cannot remember injuring it in anyway. While taking her history you learn that she recently returned from a trip overseas. What do you think is wrong with her leg? A: Stress fracture of her fibula B: Torn achilles tendon C: Deep vein thrombosis D: Arterial thrombosis

c) discoid meniscus

A college football player comes in with knee pain after a big homecoming game. The coach took him out at halftime because he knew that the athlete should not keep playing with the injury he suspected. His assumption is that the player has a closed cleavage tear, which is most common in what area of the knee? a) lateral collateral ligament b) medial collateral ligament c) discoid meniscus d) patellar tendon

A. Genu Valgum

A female patient presents to the clinic. You are observing the patients stature and notice she has large, wide hips. When the appointment is over, you dictate your note and document her large Q angle. What is this deformity called if her Q angle is larger than 17 degrees? A. Genu Valgum B. Genu Varum C. Pes Planus D. Talipes Equinovarus

A) Transverse fracture of the proximal tibial metaphysis

A four year old patient presents to the ED with pain in his knee/shin region. His mother states he was jumping on the trampoline with his older brother when he was "double-bounced." What type of fracture would possibly be seen on x-ray? A) Transverse fracture of the proximal tibial metaphysis B) Oblique fracture of proximal fibula C) Transverse fracture of patella D) Comminuted fracture of medial condyle of femur

b. Tibialis Anterior m.

A morbidly obese 23-year-old male presents to urgent care for right leg pain. He reports that he led a sedentary lifestyle up until he was recently diagnosed with type 2 diabetes. To better control his diabetes he has been going on long walks. However, since starting these walks he has been experiencing pain localized to the distal two thirds of his tibia. He has noticed edema to this area as well. The PA is highly suspicious of shin splints. Which muscle is affected? a. Fibular m. b. Tibialis Anterior m. c. Piriformis m. d. Sartorius m. e. Gastrocnemius m.

c. Genu Varum

A mother and her 4 year old son present to the clinic. The mother is concerned about her son's legs. She says that he has always been able to walk pretty normal but lately she has noticed that he tends to turn his knees inward while standing or "bow legged." You measure the Q angles of both legs and discover that they are 19 degrees. What is this condition called? a. Angle of Wiberg b. Trendelenburg Sign c. Genu Varum d. Genu Valgum e. ACL rupture

D. Sciatic nerve

A patient comes by ambulance to the trauma ER after falling sharply on a stick that pierced the medial aspect of his buttocks. You can see the wound where he pulled the stick out from and it appears deep. His leg is now mostly paralyzed. What nerve did the patient likely injure? A. Tibial nerve B. Deep fibular nerve C. Superficial fibular nerve D. Sciatic nerve

c) Piriformis

A patient comes into the clinic complaining of buttocks pain that radiates down the leg. He says that the pain is also constant and sometimes it feels like his back spasms and he can barely stand the pain. You assume that he may have a compressed sciatic nerve. What muscle is most likely associated with the compression of the sciatic nerve? a) Gluteus maximus b) Gluteus minimus c) Piriformis d) Obturator internus

B. Knee Joint, compromised blood supply

A patient comes into the clinic limping with extreme pain in her L leg. She explains that she fell forcefully on her leg while walking on the icy sidewalk. You send her for Xray and determine that she has fractured her distal femur and has separation of the condyles. What joint is likely effected by this type of fracture and what is a main concern? A. Hip Joint. compromised blood supply B. Knee Joint, compromised blood supply C. Hip Joint, nerve damage D. Knee Joint, nerve damage

b. Common fibular nerve

A patient presents to the ED with difficulty walking. On exam, she was found to have paralysis of all muscles in the anterior and lateral compartments of her right leg and while testing her gait she had an obvious footdrop. Which nerve was most likely injured? a. Tibial nerve b. Common fibular nerve c. Sciatic nerve d. Saphenous nerve

a. Fracture of Talar Neck

A patient presents to the ER following a head-on collision where they were pressing very hard on the brake pedal and now they are complaining of pain in the foot and ankle region. Knowing that the mechanism of injury involved a high-energy injury and forced dorsiflexion with axial load; what is the most likely diagnosis? a. Fracture of Talar Neck b. Fracture of Tibia c. Fracture of Medial Malleolus d. Dislocation of Metatarsophalangeal Joint

D. Biparte patella bilaterally from abnormal ossification

A patient presents to the clinic after "banging their right knee hard." They have mild to moderate pain, slight swelling of the knee and a bruise forming. You send them for Xray to be sure they have not fractured their R patella. After obtaining the results, it looks as if the R patella bone is fractured but something does not feel right about this diagnosis to you. You decide to perform another Xray, BUT this time on the L knee to compare. Results come back and the L patella looks like it has the exact same fracture as the R patella. What is the most likely reason for these results? A. Fusion of the femur and patella bone bilaterally B. Absence of patella bones C. Bilateral patella fractures D. Biparte patella bilaterally from abnormal ossification

c. Femoral nerve

A patient presents to the clinic for an annual physical. The PA performs a patellar tendon reflex during her physical exam. Which nerve is she testing the integrity of? a. Radial nerve b. Sciatic nerve c. Femoral nerve d. Superior gluteal nerve

d. Sciatic nerve

A patient presents to the clinic for treatment of multiple sclerosis. Unfortunately, the PA couldn't recall where to give an intramuscular injection in the gluteal region and rather than checking with her supervising physician she went ahead and gave the injection in the most prominent part of the patients buttock. What nerve(s) may be affected by this? a. Superior clunial nerves b. Middle clunial nerves c. Superior gluteal nerve d. Sciatic nerve e. Pudendal nerve

A . weakness of the lumbrical and interosseous muscles

A patient presents to the clinic with a foot deformity. A callus has developed on their left, 2nd digit toe. their proximal phalanx is permanently and markedly dorsiflexed (hyperextended) at the MTP joint and the middle phalanx is strongly plantarflexed at the PIP joint. The digit has a "hammer toe" appearance. What is often the cause of this deformity? A . weakness of the lumbrical and interosseous muscles B . weakness of the tibialis anterior C . stubbing one's toe really severely D . a congenital abnormality

a. 3cm inferior to the midpoint of the line joining the PSIS and the superior border of the greater trochanter

A patient presents to the clinic with pain, pallor, paresthesias, and pulselessness in his right lateral thigh. You quickly determine he has compartment syndrome and need to operate. The doctor asks you to place the anesthetic block of the sciatic nerve while he scrubs in. Where do you place the needle for the injection? a. 3cm inferior to the midpoint of the line joining the PSIS and the superior border of the greater trochanter b. 3cm superior to the midpoint of the line joining the PSOS and the superior border of the greater trochanter c. 2cm inferior to the inguinal ligament d. 2 cm superior to the inguinal ligament

A) Compartment Syndrome, fasciotomy

A patient presents to you with immense pain in his leg. When you examine the leg, you cannot find any pulses distal to the site of pain and his lower extremity is cool to the touch. You find out he was in a car accident yesterday. What is the most likely diagnosis for what he is experiencing today? How is it treated? A) Compartment Syndrome, fasciotomy B) Compartment Syndrome, amputation C) DVT, heparin D) Peripheral Arterial Disease, angioplasty

b. fracture of talar neck

A person pressing extremely hard on the break of a vehicle will most likely cause which fracture? a. comminuted fracture of calcaneus b. fracture of talar neck c. fractured metatarsals d. fracture of sesamoid bones

C. DVT

A young female patient comes into the clinic complaining of L sided leg pain and swelling. You perform a peripheral vascular exam on her and confirm the swelling, and find warmth with erythema. While conducting your interview with her, she tells you she currently is taking birth control medication and recently got back to the United States from a trip to India after taking a long 12 hour flight. With her symptoms and this information in mind, what might you suspect is causing her swollen lower extremity? A. Fractured tibia B. Varicose Veins C. DVT D. Anaphylactic Reaction

b. Posterior Hip Dislocation

A young female presents to the ED via ambulance following a high energy car accident. The paramedic says that they removed her from the vehicle where it appears the dashboard had forcefully impacted her flexed knee. (She was seated; her hip was flexed and adducted.) She is complaining of a lot of pain. This is the X-ray that is obtained, what is the most likely diagnosis? a. Sacral Fracture b. Posterior Hip Dislocation c. Anterior Hip Dislocation d. Osteoarthritis

a. Patellar tendon Patellar fracture can result from a direct blow to the knee or sudden contraction of the quadriceps muscle (ie. when someone slips and tries to prevent a backward fall)

A young male presents to the ER following a direct blow to the knee. His father explains the mechanism of injury and says that the opposing team kicked the boy directly in the knee upon missing the soccer ball. What is the most likely diagnosis? a. Patellar fracture b. Talar fracture c. Tibial fracture d. Fibular fracture

a. Epiphysial Plates

A young teenager presents with his mother to your clinic. He is complaining of knee pain and the mother notes that he has grown significantly in height over the last year. You explain that his bones are growing faster than the muscles attached to it which is causing pain. What discs made of hyaline cartilage permit the bone to grow longer? a. Epiphysial Plates b. Intervertebral discs c. Bursae d. Muscle

C) piriformis

After losing a bike race to Doofy Derek, Tiny Tanner began training for hours a day. After months of training and making a ton of gains, Tanner challenged Derek to another race. The day before the big race, Tanner developed horrible sciatic nerve pain. Hypertrophy of which muscle could lead to a syndrome of sciatic nerve pain in athletes? A) gluteus Maximus B) gluteus mediums C) piriformis D) obturator internus

b) Saphenous nerve FYI: a. ilio-inguinal nerve = femoral branch supplies skin over the medial femoral triangle c. Sural nerve = skin of posterolateral leg and lateral margin of foot d. Superficial fibular nerve = Skin of anterolateral leg and dorsum of foot, excluding web between great and 2nd toes

Amy presents at the clinic today for a regional nerve block of the lower limbs. During the procedure, Amy states that she can feel some tingling and burning that radiates to the knee and over the medial side of the leg. Which terminal branch of the femoral is most likely affected? a) Ilio-inguinal nerve b) Saphenous nerve c) Sural nerve d) Superficial fibular nerve

A) Yes; the plantar nerves in the foot are terminal branches of a derivative of the sciatic nerve

An 18 year old male underwent an anesthetic block of the sciatic nerve during a knee surgery. Upon waking up from surgery, the patient is experiencing paresthesia radiated to his foot and is concerned. Is this a normal finding? Why or why not? A) Yes; the plantar nerves in the foot are terminal branches of a derivative of the sciatic nerve B) Yes; the sciatic nerve runs as one bundle down the complete length of lower limb C) No; the nerve block completely ends where the sciatic nerve divides into the tibial and fibular nerves D) No; the only explanation is that a different nerve must have been damaged in the surgery

b. the popliteal artery

As a PA student you have to do practicals to assess your ability to perform a physical exam. On the last practical, we practiced checking peripheral pulses. The pulse behind the knee is the most difficult to palpate because this artery is very deep. What artery is this? a. the dorsalis pedis artery b. the popliteal artery c. the posterior tibialis artery d. the femoral artery

•Anterior: Dorsiflexor (extensor) compartment •Lateral: Evertor compartment •Posterior: Plantarflexor compartment (largest compartment of the 3 compartments)

Compartments of the Leg: Anterior: ___________ Lateral: ____________ Posterior: _________

b. The Lachman test

Derek and Tanner were passing the time between lectures by throwing the football around in the parking lot. Tanner couldn't contain his excitement and cross body blocked Derek. James, using his extensive orthopedic knowledge, decided Derek may have torn his ACL. What test should be performed to see if the ACL is torn? a. The Lakhan test b. The Lachman test c. The posterior drawer test d. The Houser test

c) Grade III

Derek was at the gym the other day (very rare for him) and he decided to max out on bicep curls (5lbs). This weight was just too much for him and he felt a popping sensation in his right arm. He comes into the clinic crying uncontrollably, saying the pain in unbearable and he cannot use his right arm anymore. You believe he has a muscle strain in his bicep. What class of muscle strain is this? a) Grade I b) Grade II c) Grade III d) Grade IV

C. a charley horse - the most common site for a hematoma is in the quadriceps and a charley horse is associated with localized pain and commonly follows direct trauma.

Derek, Tanner, and James decide to play some football before volleyball on a Monday night. They get rowdy and Tanner decides to tackle Derek and ends up accidentally driving his elbow into Derek's thigh. Derek tries to stand up and his quadriceps cramps up and is beginning to form a hematoma. James correctly identifies what Derek is experiencing as: A. an overreaction B. a femoral fracture C. a charley horse D. a severed nerve

a. gracilis "Because the gracilis is a relatively weak member of the adductor group of muscles, it can be removed without noticeable loss of its actions on the leg. Surgeons often transplant the gracilis, or part of it, with its nerve and blood vessels to replace damaged muscle in the hand, for example. Once the muscle is transplanted, it soon produces good digital flexion and extension."

Diane sustained a severe injury to the muscle of her dominant right hand, and "good" digital flexion and extension was lost. After consulting with a hand surgical specialist, surgery was deemed appropriate to reconstruct and return flexion and extension function. Which of the following muscles from the thigh compartment can be removed for grafting without noticeable loss of its actions on the leg? a. gracilis b. sartorius c. adductor longus d. psoas major

b.) displacement fracture of metaphysis of proximal tibia

Doofy Derek and Tiny Tanner are jumping on a trampoline. Doofy Derek double bounces Tiny Tanner. Being the tiny tyke that he is, TT is severely jolted and immediately begins crying. He refuses to bear any weight on his leg and complains of acute proximal leg pain and tenderness on palpation. After imaging, the radiologist reports back that Tiny Tanner is suffering from a typical trampoline fracture. What does this mean? a.) displacement fracture of metaphysis of distal fiibula b.) displacement fracture of metaphysis of proximal tibia c.) spiral fracture of shaft of middle tibia d.) spiral fracture of articular surface of the distal fibula

B. Trochanteric bursitis

Doofy Derek and Tiny Tanner decided they wanted to hike at Ely's Peak. Tiny Tanner was so excited he ran down the path to Ely's Peak and rolled his ankle, leaving him unable to climb the peak up ahead. Doofy Derek decided to carry Tiny Tanner on his back and he carried him all of the way to the top of the peak. As they sat on the hill to relax, Doofy Derek noticed he had tenderness over the greater trochanter of his hip and there was pain that radiated along the iliotibial tract. This pain was evident when he resisted abduction and lateral rotation of the thigh while lying on his good side. What is he experiencing? A. Growing pains B. Trochanteric bursitis C. Hamstring strain D. Piriformis syndrome

A. Anterior superior Iliac spine

Doofy Derek cried for three days after his wicked electric scooter collision with a 5 year old girl. When the scooter rammed into his feeble hip he developed what is commonly known as a "hip pointer." You know this to be a contusion involved with which prominence of the pelvis? A. Anterior superior Iliac spine B. Posterior Superior iliac spine C. Tubercle of iliac crest D. Acetabulum E. Internal lip of the iliac crest

D. Trochanteric Bursitis

Doofy Derek has spent all day moving into his 14th floor apartment. The elevator is broken and he spent the previous day moving Tiny Tanner's stuff into the apartment next door. TT is quite useless when moving heavy objects so DD moved everything but the silverware. The repetitive climbing of stairs has caused deep diffuse pain in the lateral thigh region. What other than muscle soreness is most likely causing his pain? A. Compartment syndrome B. Hairline Fracture C. Nerve damage D. Trochanteric Bursitis

a) Fabella

Doug Dimmadome owner of the Dimmsdale Dimmadome, comes into the clinic with a complaint of a nodule behind his left knee. He is worried that it may be cancerous. He feels no pain when palpating the bump, and it is located on the lateral location of the popliteal. You decide to order an X-ray and find what looks like an addition bone articulating with his femur. What is this abnormality that is present in 3-5% of people? a) Fabella b) Accessory patella bone c) Posterior superior fibular bone d) Patella

a. Femoral nerve

During one of your general surgery rotations, a man presents with a large cyst on the back of his thigh that requires draining. He is a highly anxious man so your PA proctor decides to administer a nerve block 2cm inferior to the inguinal ligament and approximately 1.5cm lateral to the femoral artery. What nerve is she trying to block? a. Femoral nerve b. Saphenous nerve c. Obturtor nerve d. Ilio-inguinal nerve

a.Shin splints, anterior compartment syndrome

Dwight is a 45yo male who presents to the clinic with pain in his anterior lower leg. He states he is not an active person but was told by his wife Angela to get off the couch, so he decided to go on a long walk. On exam his anterior muscles are tender, painful and swollen. You explain to Dwight his sudden overuse of the muscles caused swelling and inflammation reducing the blood flow to his muscles. What does Dwight have and what is it a mild form of? a.Shin splints, anterior compartment syndrome b.Shin splints, deep fibular nerve entrapment c.Injury to Tibial nerve, Extensor Digitorum longus strain d.Extensor Digitorum longus strain, anterior compartment syndrome

e. all of the above

Dwight is a 65yo male with severe CHD who has to get a vein graft for his CABG. You inform him they will most likely remove the great saphenous vein and use it in the heart to reroute blood flow around his blockage. Dwight is confused and doesn't understand why you would specifically remove his great saphenous vein. What can you tell Dwight about the great saphenous vein that makes it the most commonly used vein graft for CABG? a. it is readily accessible b. A sufficient distance occurs between the tributaries and perforating veins so that usable lengths can be harvested c. Its wall contains a high percentage of muscle and elastic fibers d. after removal it rarely causes pain or other problems in lower extremities e. all of the above

C. Gastrocnemius strain

Ethan was playing tennis and suddenly felt stabbing pain on his calf. He went to the ER and the PA informed him that he overstretched his muscle by full extension of the knee and dorsiflexion of the ankle joint. What is most likely the diagnosis? A. Soleus strain B. Accessory soleus strain C. Gastrocnemius strain D. Achilles tendon strain

a) accessory soleus

George is a long-distance runner. In recent months, he's taken to ultra-marathons, trail-running, and aspires to become and avid thru-hiker of national scenic trails. He's training for his second attempt at the fastest-known-time completion of the Superior Hiking Trail, southbound. In recent weeks, he's been experiencing pain and tenderness to touch at the posterior aspect of his ankle, specifically, medial to the calcaneal tendon. He gets it checked out at an orthopedic urgent care, and the doctor is concerned about the ongoing pain, it's location, and history of running, especially of long-distances and on uneven terrain. Chronic compartment syndrome is suspected. He is referred on to have an MRI. The MRI demonstrates some inflammation, as suspected, in an area medial to the calcaneal tendon, however it is not the soleus muscle, but something else. The doctor informs George that he is in the 3% club of folks who have this muscle and recommends he takes a break from his rigorous training. Which muscle is the doctor referring to that is present in 3% of people and may be associated with pain and swelling during prolonged exercise? a) accessory soleus b) gastrocnemius c) soleus d) fibularis longus e) fibularis brevis

d) its removal potentiates difficulties in lower limb circulation, but the reward outweighs the risk

George recently went in for a primary care appointment after noting he had radiating chest discomfort to his throat and left arm during and after exercise. His primary care provider referred him to see a cardiologist. The cardiologist recommended an angiogram to determine if there were any coronary blockages. It was discovered that his LAD and RCA were occluded and a coronary artery bypass graft was deemed the best treatment approach after the cardiologist and the cardiothoracic surgery team consulted. The great saphenous vein is typically grafted for coronary bypass for all of the following supporting reasons, except: a) it is readily accessible b) a sufficient distance occurs between the tributaries and perforating veins so that usable lengths can be harvested c) its wall contains a higher percentage of muscular and elastic fibers than do other superficial veins d) its removal potentiates difficulties in lower limb circulation, but the reward outweighs the risk

c. Trochanteric Bursitis

Geriatric James was just in for a cardiovascular stress test. And as a new PA student, you know that a cardiovascular stress test involves walking and running on an extremely inclined treadmill. James is now complaining of tenderness that radiates from his hip to the lateral side of his thigh. What is the most likely diagnosis? a. Pulled hamstring b. Fractured ischial tuberosity c. Trochanteric Bursitis d. Pulled rectus femoris

c. Claw Toes

Harry is a 72 yo male who presents to the clinic for a diabetes check up. Harry is a type 2 Diabetic and has known nerve damage in his feet. On his foot exam you see his metotarsophalangeal joints are hyperextended and his distal interphalangeal joints are flexed. You also note some corns forming on the dorsal surface of his toes. What is the name for Harrys diagnoses? a. Hammer Toes b. Clubfoot c. Claw Toes d. Metotarsalphalangeal cysts e. Bunions

c) Anterior side has tibial tuberosity

How can you tell which part of the tibia bone is the anterior surface? a) Anterior side has the lateral malleolus b) Anterior side has the soleal line c) Anterior side has tibial tuberosity d) Anterior side has the anterior border e) You cannot and must guess

d) pain or numbness along the medial border of the foot

If the saphenous nerve is injured during a saphenous cutdown. What symptoms would the patient experience? a) pain or numbness along the lateral border of the fibula b) Pain or numbness along the medial border of the fibula c) pain or numbness along the lateral border of the foot d) pain or numbness along the medial border of the foot

b) Tibialis posterior tendon

If you perform a resisted inversion test of the ankle, and there is pain, what tendon is most likely injured? a) Peroneal tendons b) Tibialis posterior tendon c) Quadriceps tendon d) Achilles tendon

a. Dorsiflexion

If you were to lift your foot up to your knee, with your toes upward, what action would this be? a. Dorsiflexion b. Plantarflexion c. Eversion d. Inversion

b. Deep lymph nodes

In the attached picture, what is number 2 pointing to? a. Superficial lymph nodes b. Deep lymph nodes c. Popliteal lymph nodes d. Axillary lymph nodes

B. Lateral Compartment

In which compartment of the lower leg are these structures located? Superficial fibular nerve, fibularis brevis, and fibularis longus. A. Anterior Compartment B. Lateral Compartment C. Posterior Compartment D. Medial Compartment

D. Paralysis of Quadriceps

Jamie is a 55yo female who presents with severe knee pain. She states she was in a car accident and her left knee slammed into the glove compartment of the car. On exam you notice she is walking with a forward lean and her hand is placed on her distal thigh. After further examination you observe that she is unable extend her left leg against resistance. What is a potential diagnosis for Jamie? A. Fibular fracture B. Paralysis of Gracilis C. Femoral Fracture D. Paralysis of Quadriceps

D on the medial side of the foot On the medial side of the foot, passing superior to the abductor hallucis to allow visualization of critical neurovascular structures, while avoiding production of a painful scar on a weight-bearing area

Jane returned from vacation in Florida, where she was walking barefoot on the beach the entire time. She presents to your clinic with swelling and pain in her left foot, indicative of a foot infection. If this infection is localized within the compartments between the muscular layers, where should you try to make an incision to drain it? A on the lateral side of the foot, B on the dorsum of the foot. C in the ankle D on the medial side of the foot

a) deep vein thrombosis

Jane runs a nonprofit that has has been heavily involved in helping with ongoing relief efforts in San Juan, Puerto Rico and the US Virgin Islands since Hurricane Maria hit in September of 2017. She travels frequently via airplane between these locations and back home to Minnesota. Between the work on the ground and being mobile and running the nonprofit on-the-go, she takes advantage of the longer flights to rest and catch up on sleep, mostly staying in her assigned seat during the flights. On her most recent return trip home, she was redirected to two additional stops and connecting flights due to weather. Several days after arriving home in Minnesota, she began experiencing pain in her lower right leg, and noted it was warm to the touch. Given her travel history described, what is the most likely diagnosis? a) deep vein thrombosis b) hypokalemic c) gastrocnemius strain d) shin splints

c) medial

Jane, a 35 year old female is getting plastic surgery on her buttocks. She states she wants a lift done to the superior, inferior, lateral, and medial portions. During surgery, a new resident accidentally nicked the sciatic nerve, leaving Jane with paralysis of the semitendinosus, semimebranosus, and biceps femoris. In addition Jane now has impaired thigh extension and leg flexion. What area of the buttock did this injury likely take place? This area is also known as the side of danger. a) superior b) inferior c) medial d) lateral

C. Compound fracture

Jeremy was walking across the street when he was hit by a car. The car stopped, but not before the bumper of the car struck his leg. An x-ray revealed a fracture of the tibial shaft, which you know is the most frequent site of fracture of the tibia. The tibial shaft is also the most common site for what type of fracture that you would suspect in this case of "bumper fracture"? A. Spiral fracture B. Transverse "boot top" fracture C. Compound fracture D. Spiral fracture E. Transverse march fracture

b) calcaneal tendon - ruptured

Jerry, a 45 y/o males presents to your clinic with a 1 day history of calf and ankle pain. He states he got together with some high school friends last night. His friend who used to be in track with him bet Jerry that he could still beat him in a sprint. Jerry, who hasn't worked out in 5+ years, decided this was a good idea. During his forceful push off to begin his sprint he heard an audible snap which was immediately followed by sudden calf pain and dorsiflexion of his plantarflexed foot. there is not a palpable 3 cm gap on the back of his ankle. What is the structure he most likely injured, and how? a) gastrocnemius - strain b) calcaneal tendon - ruptured c) ACL - tear d) tibialis anterior - strain (shin splints)

b) Anterior Cruciate Ligament

Jessica injured her knee during a soccer game. Upon an MRI you notice that the weaker of the Cruciate ligaments was torn. Which ligament is this? a) Posterior Cruciate Ligament b) Anterior Cruciate Ligament c) Lateral Cruciate Ligament d) Medial Cruciate Ligament

a. piriformis

Jill, a 20 year old ice skater, presents to the clinic with pain in her left buttock. She does not remember any falls or injuries that could have caused this pain. She does state that she has been lifting more frequently at the gym and is working on strengthening her gluteal muscles as her skating career is getting serious and she is looking forward to participating in the next winter Olympics. You inform her that she most likely has injury of the sciatic nerve due to compression by which muscle? a. piriformis b. superior gemellus c. obturator internus d. quadratus femoris

c) imaging of the other knee; the defect will be bilateral

John is a 43 y/o male who plays basketball every saturday morning with his friends. He recently came down from a rebound with his knee at a weird angle, causing a pop and extreme pain. After going to the ER and getting an MRI he was diagnosed with a torn MCL. However, during the MRI they also noted a possible patellar fracture. He states he has had that diagnosis before when he received imaging for a different reason, but has no history of pain in the area. One of the student notes that it could possibly be due to abnormal ossification of the patella. What would be your next diagnostic test to determine this, and how would you know if this is the correct diagnostic? a) blood testing for genetic abnormalities; it is a hereditary deformity b) patellar tendon reflex test; the tendon will be absent c) imaging of the other knee; the defect will be bilateral d) there are no tests for this; it is completely random

d. rickets

Johnny grew up in a very poor family in Appalachia with little to eat. When he saw a provider for the first time as a toddler he was severely malnourished and Xrays showed Coxa Vara. What underlying pathology likely caused Johnny's weak bones and deformity? a. hypercalcemia b. osteomyelitis c. osteoporosis d. rickets

d. Head of fibula

Johnny, 23-year-old male, presents to the clinic with a swollen knee and needs a direct aspiration of the knee. Which of the following bony points is NOT considered a landmark for a direct aspiration that is performed using the lateral approach? a. Anterolateral tibial (Gerdy) tubercle b. Lateral epicondyle of the femur c. Apex of patella d. Head of fibula e. All of these are considered landmarks

a. The bones are growing faster than the attached muscle

Johnny, an 11-year-old boy, comes in to see you complaining of right leg pain. He is the star of his hockey team and is worried he won't be able to keep playing. You notice his pants fit as if he's waiting for a flood and assume he must be in the middle of a growth spurt and that must be causing his pain. Why would he be feeling pain if he wasn't hit during a game? a. The bones are growing faster than the attached muscle b. The muscles are growing faster than the bones c. He just wanted to skip school today so he made it up d. The ligaments are growing faster than the bones

d) compartment syndrome

Johnny, while trying to protect his quarterback, sacrificed his body and took a direct hit to the thigh. After experiencing this blunt trauma, he reported to the clinic. While doing a peripheral vasculature exam, you notice loss of distal leg pulses. What is Johnny most likely suffering from? a) varicose veins b) thrombosis c) saphenous nerve injury d) compartment syndrome

d) shortening of the triceps surae resulting in a lump in the calf Reasoning: Gastrocnemius, soleus, and plantaris muscles ARE affected. Rupture is often sustained by poorly conditioned people with a history of calcaneal tendinitis. Individuals cannot raise their heel from the ground or balance on the affected side.

Justin, a 42 year old male, presents to your clinic with bruising over the malleolar region and difficulty walking. He states that he was tired of his sedimentary lifestyle and started training for Grandma's marathon. On one of his training runs, he heard an audible snap followed by sudden calf pain and dorsiflexion of the plantarflexed foot. Diagnosis of ruptured calcaneal tendon was made. What other finding would most likely be noted with this diagnosis? a) unaffected gastrocnemius, soleus, and plantaris muscles b) no history of previous calcaneal tendinitis c) ability to lift heal off the ground while standing d) shortening of the triceps surae resulting in a lump in the calf

c. Complete rupture of the muscle fibers

Krazy Kelsey was working out at the gym. She went to the leg extension machine, loaded it up to 215 lbs, and performed a single leg extension. Right before she extended her leg entirely, she felt a large "pop" with immediate pain. She went to the ER where the PA diagnosed her with a Grade III rectus femoris muscle injury. What does a Grade III muscle injury consist of? a. Only a limited number of muscle fibers damaged b. 50% of the muscle fibers are damaged c. Complete rupture of the muscle fibers d. Necrosis of the muscle

a. Hallux valgus

Lackadaisical Lauren comes into the clinic with new foot pain. A radiograph is reviewed of the right foot and demonstrates medial deviation of the 1st metatarsal. The great toe is also overlapping the 2nd toe on examination. She is unable to move her great toe away from the 2nd toe due to the lateral shift of the sesamoids. What is her diagnosis? a. Hallux valgus b. Hallux varus c. Claw toe d. Pes planus

A) Pes Planus (flat feet)

Lately Anna has been experiencing pain in her feet after she rapidly started to gain weight. You ask her to stand and it looks like she has fallen arches. You take some imaging and notice that her plantar calcaneonavicular ligament looks like its failing to support the head of the talus, therefore causing the talus to displace inferomedially. What would you diagnose Anna with? A) Pes Planus (flat feet) B) Talipes Equinovarus (Clubfoot) C) Claw Toes D) Hammer Toe

D. A &B -- extension of the hip & flexion of the leg

Lauren, a 19 year old ice skater, presents to the clinic after falling on her buttocks during practice. Your exam reveals complete section of the sciatic nerve. What movement impairment(s) would you expect in this injury? A . extension of the hip B . flexion of the leg C . extension of the leg D . A & B

A . plantarflexion of the ankle joint

Lauren, a 20 year old female, presents to the clinic for a routine physical. You, the PA, want to test her calcaneal tendon reflex. What reaction would conclude a normal result? A . plantarflexion of the ankle joint B . plantarflexion of the big toe C . no reaction D . dorsiflexion of the ankle joint

A . ossification; "riders' bones"

Lauren, a 20 year old horseback rider presents to the clinic for pain in her inner thighs. You conclude that she has muscle strains of her adductor longus. What is a potential complication to the tendons of these muscles and what are they sometimes (wrongly) called? A . ossification; "riders' bones" B . softening; "rider's tendons" C . ossification; "football bones" D . degeneration; "ballerina's foot"

A . she fractured one of her metatarsals (dancer's fracture)

Lauren, an 18 year old ballerina, presents to the clinic after losing her balance at ballet practice while working on her demi-pointe technique. Her left foot is in severe pain and she says that when she lost her balance, all of her bodyweight was on the middle part of her foot, pointing to her metatarsals. What do you suspect happened? A . she fractured one of her metatarsals (dancer's fracture) B . she sprained her ankle (dancer's ankle) C . she pulled her calf muscle (dancer's calf) D . she fractures on of her metacarpals (dancer's fracture)

b. Coxa Vera

Lilly is a 1 yo female who just started walking. Her mother brought her to the clinic because she noticed Lilly is walking with a sort of limp and is concerned. After examination you can clearly state her left leg is shorter than her right and she has limited passive abduction of her hip. You tell her mother the cause could be due to a congenital defect in which her the femoral head ossified incorrectly causing the shaft of the femur to deviate toward midline. What is another term to describe Lillys condition? a. Coxa Valga b. Coxa Vera c. Slipped upper femoral epiphyses d. Head of femur fracture

A. Great saphenous vein

Lily is only one month old but was born with tetralogy of Fallot and needs to have open-heart surgery. The anesthetist is most likely going to use ultrasound to find which vein lateral to the medial malleolus since it is a reliable point for IV access in infants and children? A. Great saphenous vein B. Small saphenous vein C. Anterior tibial vein D. Posterior tibial vein E. Fibular vein

c) Weakness in the epiphysial plate causing dislocation

Madeline a 13 year old girl comes into the clinic because she has been experiencing hip discomfort that is referred to the knee. She experiences the pain when she laterally rotates her thigh and abducts her thigh. You need radiographic evidence to confirm your diagnosis. What diagnosis are you thinking: a) Femoral Fracture b) Torn sacrotuberous ligament c) Weakness in the epiphysial plate causing dislocation d) Torn sacrospinous ligament

c) Talar neck fracture

Maniac Megan was driving to class one day after hitting up the local bar. Not knowing the light turned red in front of her, she swerved to miss the car that was stopped. Instead, she hit another vehicle in a head on collision. Her right ankle was dorsiflexed extremely hard. What fracture is most likely to be seen in a severe dorsiflexion injury? a) Calcaneal fracture b) Os trigonum c) Talar neck fracture d) Middle cuneiform fracture

D. Accessory obturator artery

Marie is a 42-year-old female who has had six children and is now having surgery to repair a strangulated femoral hernia. While the surgeon is placing staples during the endoscopic repair, what common arterial variant that runs close to or across the femoral ring should they be aware of? A. Accessory femoral artery B. Accessory profunda femoris artery C. Accessory inferior epigastric artery D. Accessory obturator artery E. Accessory circumflex femoral artery

B. Clubfoot

Mary just had a baby boy. She notices her baby's feet were inverted, ankle plantarflexed and forefoot is adducted. What is the most likely diagnosis of Mary's baby? A. Pes Planus B. Clubfoot C. Claw Toes D. Hammer Toes

b. patellofemoral syndrome

Michael Scott has been running several days a week in order to train for the upcoming 5K race supporting rabies awareness. He presents to your office complaining of pain deep to his patella which he notices especially when he is running downhill. After performing a thorough history and physical exam, you believe Michael has what is sometimes referred to as "runner's knee". Based on the above information, what is Michael's most likely diagnosis? a. patellar dislocation b. patellofemoral syndrome c. torn ACL d. popliteal cyst

A. Plantar Fasciitis

Neymar, a Brazilian soccer player presents to medical staff with pain on the plantar surface of his foot. He often trains barefoot and he notices the pain when starting exercise after periods of rest. His pain is the worst in the morning during the first activity after waking up. His pain increases when the trainer passively extends the great toe. What is the source of Neymar's pain? A. Plantar Fasciitis B. Sprain C. Calcaneus fracture D. Cellulitis

a) Medial to femoral artery/ 3cm inferior to the inguinal ligament

On your first day of a new clinical rotation in vascular surgery, you are excited to observe a varicose vein operation. The operation will require tying off the great saphenous vein. Your preceptor reminds you that in some people the femoral vein can be superficial and these two veins can be confused. They ask where you would expect to find the femoral vein and great saphenous vein respectively. a) Medial to femoral artery/ 3cm inferior to the inguinal ligament b) Lateral to the femoral artery/ 3 cm inferior to the inguinal ligament c) 3cm inferior to the inguinal ligament/ Medial to the femoral artery d) 3cm inferior to the inguinal ligament/ lateral to the femoral artery

B) trochanteric bursa

One of your longtime patients comes into the clinic and to your surprise, they have lost a significant amount of weight (>10%) after being overweight for many years. To rule out risk of malignancy, you ask if they have made any lifestyle changes to explain this weight loss. They state that they have been running up and down stairs and doing hill sprints for exercise. They are now suffering significant discomfort over their lateral hip any time they try to step up with that leg. Their range of motion is normal, and you believe that inflammation is a culprit. What structure is most likely affected? A) ischial bursa B) trochanteric bursa C) hamstring D) sciatic nerve

B. superficial veins; deep veins

Perforating veins deliver deoxygenated blood from the ___________ to the ________________. A. deep veins; superficial veins B. superficial veins; deep veins C. superficial veins to lymph system D. lymph system to superficial veins

d. fanning of lateral 4 toes and dorsiflexion of great toe; brain injury or cerebral disease

Plantar reflex was tested on a 20 year old male and the Babinski sign was noted and documented. What was the reaction and what does that reaction indicate? a. flexion of toes; muscle injury b. flexion of toes; brain injury or cerebral disease c. fanning of lateral 4 toes and dorsiflexion of great toe; muscle injury d. fanning of lateral 4 toes and dorsiflexion of great toe; brain injury or cerebral disease

e) Both A & B Baker's and Popliteal cysts

Prince Bartholomew Worthington Uppercrust IV was riding his prize horse Sir Clippidy Cloppidy and started to notice a small lump behind his left knee. The lump has been there for about a month now and he feels no pain but he says he can feel it when he flexes his leg and is somewhat bothersome. What most likely is the problem Prince BWU 4th is having? a) Baker's cyst b) Popliteal cyst c) Patellar cyst d) Both B & C e) Both A & B

C. Calcaneal Tendinitis

Runner Ray presents to your clinic complaining of pain in the back of his foot and leg while walking in his dress shoes at work. He states he just took the last few months off after finishing his last marathon. A few days ago he just restarted training for another marathon his friend pressured him into on short notice. Given that his pain is presenting following the new start up of his vigorous marathon training and is worse when walking in his rigid-soled dress shoes at work, what is the most likely diagnosis? A. Tibial Fracture B. Ischial Bursitis C. Calcaneal Tendinitis D. Contusion of Extensor Digitorum

b) rapid growth

Rupert, a 7 year old boy, wakes up and tells his mom that he had an irritating feeling in his leg while trying to sleep. "My gosh Rup!", his mom said, "you must have grown 2 inches over night!" What most likely caused the irritating feeling Rupert is complaining of? a) sleep walking b) rapid growth c) femur fracture d) avulsion fracture

d) Saphenous Varix "A localized dilation of the terminal part of the great saphenous vein, called a saphenous varix, may cause edema in the femoral triangle. A varix should be considered when varicose veins are present in other parts of the lower limb."

Sally, a 55 year old female, presented to the clinic today with edema in the femoral triangle. She insists that it is an abscess, because that is what her friend had once upon a time, and it looks very similar. Upon examination of her leg, many varicose veins are noted. Along with a psoas abscess, what is most likely to be added to the top of your differential diagnosis list? a) Thigh contusion b) Pulled groin c) Injury to adductor longus d) Saphenous Varix

b) neck of the femur

Stacy, a 46 y/o women comes in to the ED after falling down her stairs. She states that she is excruciating pain in her left thigh. After looking at her chart, you notice she was diagnosed with osteoporosis last july. You get an X-ray and notice a fracture to her femur. What part of the femur is most likely broken given her history and the anatomy of the bone? a) shaft of the femur b) neck of the femur c) greater trochanter d) lesser trochanter

a. Cannot extend leg against resistance

Stanley, 61-year-old male, presents to the clinic walking with a forward lean, pressing on the distal end of his thighs with his hands. This adaptive gait prevents inadvertent flexion of the knee joints. Paralysis of the quadriceps is presumed. What test would support this suspicion? a. Cannot extend leg against resistance b. Cannot flex leg against resistance c. Cannot abduct leg against resistance d. Cannot adduct leg against resistance e. There are no tests to confirm this suspicion.

d) tibialis posterior

Superficial muscles of the posterior compartment of the leg include each of the following, except: a) soleus b) plantaris c) gastrocnemius d) tibialis posterior

D) Hallux Valgus

Tanya and Dereka decided they wanted to spice up their wardrobe for graduate school and started wearing high heels to class everyday. The pressure and friction from those shoes resulted in tender bunions. They pushed through the pain because "beauty is pain," but then their friend Jamie pointed out that their great toes are deviated laterally. What is this foot deformity called?' A) Pes Planus B) Baker Cysts C) Claw Toes D) Hallux Valgus

a. quadriceps muscles; femoral nerve

Tapping the front of the knee elicits a "knee-jerk" or kick like reaction. This is often done in clinic during physical exams. What muscles fibers contract in response to patellar tapping ? What nerve integrity is being tested? a. quadriceps muscles; femoral nerve b. hamstring muscles; sciatic nerve c. calf muscles; tibial nerve d. patellar muscles; patellar nerve

c. hamstring

Terry is a 16yo male who presents to the clinic with severe pain in his right leg. He states he was playing in his soccer game when he suddenly felt the pain. He notes he was late to warm ups and was not able to fully stretch like he usually does. On exam when fully extending Terrys right leg he nearly falls over in pain. Considering Terrys severe pain when extending his leg, the fact that he missed warm ups, and how common tearing this muscle is what muscle did Terry most likely tear. a. quadricep b. rectus femoris c. hamstring d. gluteus maximus

A) PCL

The CSS PA program intramural volleyball team were at their final game night of the season, watching the teams before them finish up their match. One of the players on the court appeared to trip over a teammates foot, but was unable to get back up. Before the AT or PT program could get involved, the PA students jumped in to assess the player. Having determined that the injury occurred in the flexed knee with a twisting motion, what structure is not part of the classic "triad" for this injury? A) PCL B) TCL C) medical meniscus D)ACL

d) nerve to the vastus intermedius "The adductor canal provides an intermuscular passage for the femoral artery and vein, the saphenous nerve, and the slightly larger nerve to the vastus medialis, delivering femoral vessels to the popliteal fossa where they become popliteal vessels."

The adductor canal provides an an intermuscular passage for each of the following, except: a) femoral artery and vein b) the saphenous nerve c) nerve to vastus medialis d) nerve to the vastus intermedius

A. Flexion of the toes

The doctor you are doing a clinical rotation with asks you to performing a neurological examination on the adult, male patient in the next room. You decide to check his plantar reflex- as this is commonly performed during neurological examinations. You stroke the lateral aspect of his foot with a blunt object to perform the test. What would indicate a normal response? A. Flexion of the toes B. Flexion of the lower leg C. Extension of the lower leg D. Fanning of the toes

a. femoral nerve

The femoral sheath is a continuation of the transversalis fascia of the abdomen. Which of the following femoral triangle contents is not enveloped by the femoral sheath? a. femoral nerve b. femoral artery c. femoral vein d. femoral lymphatics

a. muscle attachment "The fibula has no function in weight bearing. It serves mainly for muscle attachment ... ".

The fibula's main function is: a. muscle attachment b. weight bearing c. stabilize the patella d. fully form malleolus

a. it has a greater tubercle

The following are all true regarding the femur EXCEPT: a. it has a greater tubercle b. it is the longest bone in the body c. it is the heaviest bone in the body d. it has an intercondylar fossa

C) gluteus mediastinum

The gluteal muscles attach proximally to the gluteal lines of the ala of ilium and attach at the greater trochanter and lateral condole of the tibia. Which of these is not a real gluteal muscle? A) gluteus Maximus B) gluteus medius C) gluteus mediastinum D) gluteus minimus

A. Dorsal vein of the big toe & Dorsal venous arch Dorsal vein of little toe and dorsal venous arch form the small saphenous vein

The great saphenous vein is formed by which 2 veins? A. Dorsal vein of the big toe & Dorsal venous arch B. Dorsal vein of the little toe & Dorsal venous arch C. Dorsal venous arch & anterior tibial vein D. Dorsal venous arch & profunda femoris vein

c. Ilium, Ischium and Pubis

The hip bone is made up of the fusion of what 3 bones? a.Ischium, Pubis, and coccyx b. Ilium, Pubis and Coccyx c. Ilium, Ischium and Pubis d. Ilium, Ischium and Coccyx

A. Tibial Nerve

The semitendinosis muscle assists in the action of extending the thigh and flexing the knee. Which nerve innervates the semitendinosus muscle? A. Tibial Nerve B. Sciatic nerve C. Fibular nerve D. Saphenous nerve E. None of the above

d. axial line axial line: line of junction of dermatomes that are supplied from discontinuous spinal levels, found along the back of the leg, from right under the butt cheeks down to the ankle.

The skin of the lower limb is innervated by cutaneous branches of the lumbar and sacral plexuses. The segments of skin innervated by these nerves are called dermatomes and they overlap significantly EXCEPT for where?? a. anterior dermatome line b. posterior dermatome line c. transverse line d. axial line

b) Medial and Intermediate Cutaneous Nerves

The superficial fibular nerve passes through the deep fascia to become a cutaneous nerve which divides into what two nerves? a) Lateral and Medial Cutaneous Nerves b) Medial and Intermediate Cutaneous Nerves c) Superior and Inferior Cutaneous Nerves d) Intermediate and Lateral Cutaneous Nerves

a) gluteus medius, gluteus minimus, and tensor fasciae latae muscles

The superior gluteal nerve is responsible for innervation of which muscles? a) gluteus medius, gluteus minimus, and tensor fasciae latae muscles b) gluteus medius, gluteus minumus, gluteus maximus c) gluteus maximus and tensor fasciae latae muscles d) tensor fasciae latae muscles

b) Clubfoot

There is a congenital abnormality of the subtaler joint that often results in inversion of the foot, plantarflexion of the ankle, and adduction of the forefoot. The foot will assume the position of a horse's hoof. An affected patient will be unable to put the heel and sole flat and must bear the weight on the lateral surface. What is this called? a) Claw toes b) Clubfoot c) Pes Planus d) Hammer toe

the tendon may not be able to provide stability and support for the arch of the foot, resulting in flatfoot ACTION: Plantar flexes ankle/ INVERTs foot

Tibialis posterior What can occur if there is posterior tibial tendon dysfunction?

D. Charley Horse

Tiny Tanner is the smallest guy on his hockey team, and unfortunately for him, seems to get hit the most. Last night he was slashed in the thigh by the opposing team and has been complaining of localized pain on his anterior thigh. What is the commonly used term to describe what Tanner is feeling? A. Jack Russel Terrier B. Davey Horse C. Charlie Sheen D. Charley Horse

c. heat

Tiny Tanner limps into the clinic and complains of a charley-horse from an opponents helmet slamming into his anterior thigh during football practice earlier in the day. You diagnose him with a grade II muscle hematoma. What should he NOT do for the leg injury? a. rest b. ice c. heat d. elevate e. compression

B. Compartment syndrome

Tiny Tanner was playing in the playplace area at his favorite neighborhood McDonald's. His mother yelled to him that his 2 Big Macs, McChicken, fries, and apple pie were ready for lunch. Tiny Tanner was overcome with joy and excitement. He took quite the tumble from the top of the jungle gym and hit his thigh on a metal beam. After self medicating with a McFlurry, TT noticed excessive bruising and felt excruciating pain in his leg. When you see TT in the ER, you immediately check distal pulses and find none. You also notice the injured leg is cool to the touch. What diagnosis do you fear for TT? A. Thrombophlebitis B. Compartment syndrome C. Saphenous Nerve injury D. Lower leg Fascitis

d) Calcaneal Tendinitis

Tiny Tanner was tiny in his earlier years but became not so tiny in the later years, gaining over 300lbs. He recently started a walking program after years of a sedentary lifestyle and lost quite a bit of weight. When he started running as part of the program he started to complain of heel and foot pain. He comes into the clinic to discuss his pain. You notice that he is wearing toe shoes. You ask him what shoes he uses to run in for his program and he points to his toe shoes. With your knowledge of running injuries, that arise from poor footwear and sudden repetitive impact from years of inactivity can result in what injury? a) Calcaneal Rupture b) Tibial Nerve Injury c) Fracture of the Calcaneus d) Calcaneal Tendinitis

b. Sciatic, superolateral quadrant of buttocks

Tiny Tanner went to the clinic to get an antibiotic shot for an infection. Unfortunately, the PA gave Tiny Tanner the intragluteal injection at the most prominent part of his buttocks. As a result, Tiny Tanner suffered from nerve damage. Which nerve was likely injured and where should the PA have injected the shot had they been practicing proper technique? a. Sciatic, posteromedial quadrant of buttocks b. Sciatic, superolateral quadrant of buttocks c. Gluteal, anterolateral quadrant of thigh d. Gluteal, posteromedial quadrant of buttocks

b. calcaneal bursitis FYI: Calcaneal Tendinitis could also be on the differential because it occurs during repetitive activities such as running, especially after long periods of inactivity. A ruptured tendon would be so much more severe and the patient probably wouldn't be able to walk. Also, an audible snapping may be heard and there may be a palpable gap on the posterior heel if it is a completely ruptured tendon. Gastrocnemius strain would cause stabbing pain in the calf and typically occurs in people over 40 years old.

Tom and Derek were playing Tanner and James in a little game of pick up basketball. After the game, Derek kept complaining of heel pain when walking. James suggested a diagnosis based on the excessive and repeated friction on Derek's heel from running up and down the basketball court. What is this diagnosis? a. ruptured calcaneal tendon b. calcaneal bursitis c. gastrocnemius strain d. calcaneal fracture.

Fx in left proximal tibial metadiaphysis

Trampoline fracture

1. Extensor Digitorum Muscle 2. Patellar Ligament 3. Tibia 4. Popliteus Muscle 5. Great Saphenous Vein 6. Gastrocnemius Muscle Medial Head 7. Tibial Nerve 8. Soleus Muscle 9. Fibula 10. Pernoneus Longus Muscle

Upper portion of leg

A. Sciatic

Valerie is a professional ice skater. She had just completed her first round of the US figure skating. Afterward she suddenly started complaining of pain in her buttocks. The PA working for the US figure skating diagnose Valerie of Piriformis Syndrome. Which nerve did Valerie most likely compressed? A. Sciatic B. Clunial C. Superior Gluteal D. Obturator

D . competing in athletic events such as sprinting or jumping

Venous stasis (stagnation), an important cause of thrombus formation, is caused by ALL of the following EXCEPT: A . incompetent, loose fascia that fails to resist muscle expansion, diminishing the effectiveness of the musculovenous pump B . external pressure on the veins from bedding during a prolonged hospital stay or from a tight cast or bandage C . muscular inactivity (e.g. during an overseas aircraft flight) D . competing in athletic events such as sprinting or jumping

a. Superiorly: inguinal ligament, medially: lateral border adductor longus, laterally: sartorius

What are the boundaries of the femoral Triangle? a. Superiorly: inguinal ligament, medially: lateral border adductor longus, laterally: sartorius b. Superiorly: lateral border adductor longus, medially: sartorius, laterally: adductor brevis c. Superiorly: inguinal ligament, medially: gracilis, laterally: sartorius d. Superiorly: gracilis, medially: lateral border adductor longus, laterally: inguinal ligament

d. obturator membrane

What covers the majority of the great sciatic foraman? a. sacrospinus ligament b. sacrotuberous ligament c. illiotibial band d. obturator membrane

b) Posterior

What dislocation of the hip joint is most common? a) Anterior b) Posterior c) Lateral d) Inferior

c) Slipped upper femoral epiphysis

What does the above radiograph show? a) Broken femur b) Normal Xray c) Slipped upper femoral epiphysis d) Broken pelvis

d) All of the above

What evolved features are unique to the human foot? a) Medial migration of the distal attachment of the fibularis longus across the sole of the foot. b) Development of a fibularis tertius that is attached to the base of the 5th metatarsal c) Relatively everted (pronated) feet. (Causes sole to lie more fully on the ground) d) All of the above

d. gluteus maximus

What gluteal muscle does the inferior gluteal nerve innervate? a. gluteus medius b. gluteus minimus c. obturator internus d. gluteus maximus

A. Compression of deep fibular nerve

What is a cause of pain in the anterior compartment of the leg during skiing, running, and dancing? A. Compression of deep fibular nerve B. Compression of deep tibial nerve C. Compression of ulnar nerve D. Compression of the Achilles tendon

B. Fibular collateral ligament

What is another name for the lateral collateral ligament? A. Medical collateral ligament B. Fibular collateral ligament C. Tibial collateral ligament D. Anterior Cruciate ligament

b. Medial malleolus

What is considered your ankle bone? a. Lateral malleolus b. Medial malleolus c. Ulnar styloid process d. Radial styloid process e. Head of femur f. Patella

b) incompetent, loose fascia

What is one thing that can cause venous stasis leading to a potential DVT? a) over-exercising b) incompetent, loose fascia c) excessive water consumption d) fibrous vein walls

D. The Q angle

What is the angle between the femur and the tibia clinically referred to as? A. The Knee angle B. The patella angle C. The S angle D. The Q angle

d. Iliopsoas

What is the chief flexor muscle of the thigh? a. Gluteus Maximus b. Superior Gemellus c. Saritorius d. Iliopsoas

A. Quadriceps tendon

What is the distal attachment of rectus femoris? A. Quadriceps tendon B. Lesser trochanter of femur C. Pectinal line D. Greater trochanter of femur

D. Anterior rami of lumber nerves (L1, L2)

What is the innervation of psoas minor A. Femoral nerve B. superior gluteal nerve C. inferior gluteal nerve D. Anterior rami of lumber nerves (L1, L2)

D) Sartorius

What is the longest muscle in the body, found in the anterior compartment of the thigh, that is mainly a synergist muscle? A) Iliopsoas B) Piriformis C) Rectus femoris D) Sartorius

b. sartorius

What is the longest muscle in the body? a. gracilis b. sartorius c. adductor longus d. iliopsoas

a. Adduction

What is the main action of plantar interossei muscles? a. Adduction b. Abduction c. Extension d. Flexion

B: Lateral rotation

What is the main action of the obturator externus? A: Medial rotation B: Lateral rotation C: Extension D: Flexion

C. Infections of the foot

What is the most common cause of enlargement of the popliteal and inguinal lymph nodes (lymphadenopathy)? A. Eye infections B. Finger infections C. Infections of the foot D. Continuous McDonald's eating

B. Fabella

What is the name of the sesamoid bone, located within the gastrocnemius muscle, that is commonly found accidentally in an X-ray (similar to the one included) in just 3-5% of the population? A. Patella B. Fabella C. Fibula D. Popliteal E. Calcaneal

b. Quadriceps tendon

What is the patellar ligament a continuation of? a. Femoral triangle b. Quadriceps tendon c. Iliotibial Band d. Hamstring tendon

A. Tibialis Anterior

What is the strongest dorsiflex muscle? A. Tibialis Anterior B. Extensor digitorum longus C. Extensor hallucis longus D. Fibularis tertias

c) The lining fascia between the transversus abdominus and peritoneum.

What is the transversalis fascia? a) The compartment between the anterior and posterior portion of the upper leg. b) The lining fascia between the bladder and peritoneum. c) The lining fascia between the transversus abdominus and peritoneum. d) The compartment near femoral joint because it appears to be transverse. e) The lining fascia transversing the femur separating lower leg from upper.

B. bones in the front of your foot move out of place, causing the tip of your big toe to get pulled toward the smaller toes and forces the joint at the base of your big toe to stick out

What is the underlying cause of a bunion? A. bones in the front of your foot move out of place, causing the smaller toes to get pulled toward the tip of the big toe and forces the joint at the base of your big toe to stick out B. bones in the front of your foot move out of place, causing the tip of your big toe to get pulled toward the smaller toes and forces the joint at the base of your big toe to stick out C. bones in the back of your foot move out of place, causing your big toes and smaller toes to move forward D. bones in the front of your foot move out of place, causing the tip of your big toe to get pulled toward the smaller toes and forces the joint at the base of your big toe to to get pulled laterally

b) Sacrotuberous ligament

What ligament connects the Sacrum to the Ischial tuberosity? a) Sacrospinous ligament b) Sacrotuberous ligament c) Obturator membrane d) Lessor Sciatic Ligament

c) Inversion

What movement of the foot and ankle causes the soles of the feet to face inward and commonly results in ankle sprains? a) Dorsiflexion b) Plantarflexion c) Inversion d) Eversion

c. Ischial tuberosity

What muscle and/or structure does one have the most pressure on while sitting? a. Gluteus maximus b. Gluteus minimus c. Ischial tuberosity d. None of the above

d) Prirformis

What muscle can cause injury to the sciatic nerve via compression or spasm? a) Gluteus minimus b) Gluteus medius c) Gluteus maximus d) Prirformis

A: Fibularis tertius

What muscle in the lower leg is not present in all individuals and is thought to play a proprioceptive role in sensing sudden inversion? It is also thought to reflexively contract to protect the anterior tibiofibular ligament, the most commonly sprained ligament in the body. A: Fibularis tertius B: Flexor digitorum longus C: Fibularis brevis D: Tibialis posterior

B) Quadriceps femoris

What muscle is most important in stabilizing the knee joint? A) Gastrocnemius B) Quadriceps femoris C) Biceps femoris D) Vastus lateralis

d) Quadriceps Femoris

What muscle is the most important "stabilizer" of the knee? a) Sartorius b) Gluteus Maximus c) Hamstrings d) Quadriceps Femoris

c. Vastus Intermedius

What muscle lies directly underneath the rectus femoris mucle? a. Vastus Lateralis b. Vastus Medialis c. Vastus Intermedius d. Gracilis

B: The tibial nerve

What nerve travels between the two heads of the gastrocnemius and innervates all the muscles in the posterior compartment of the leg? A: The fibular nerve B: The tibial nerve C: The achilles nerve D: The soleus nerve

d) 20%

What percentage of the gait cycle is considered double support? a) 80% b) 60% c) 10% d) 20%

c. active adults

What population would be the most likely to be diagnosed with fatigue fractures of the metatarsals? a. infants b. sedentary adults c. active adults d. the elderly

C. Plantaris

What small muscle with a short belly and a long tendon is absent in 5-10% of people and only weakly assists the gastrocnemius in plantarflexing the ankle? A. Soleus B. Popliteus C. Plantaris D. Flexor hallucis longus E. Tibialis posterior

a. inferior part of the fossa

Where are the pulsations of the popliteal artery best felt? a. inferior part of the fossa b. superior part of the fossa c. medial part of the fossa d. lateral part of the fossa

a) ischial tuberosity

Where does all of the body's weight rest while sitting? a) ischial tuberosity b) lesser sciatic notch c) ischiopubic ramus d) auricular surface e) ramus

D ischial tuberosity

Where does the body's weight rest while sitting? A Body of illium B tubercle of the iliac crest C ischiopubic ramus D ischial tuberosity

c. Transveralis fascia

Where does the femoral sheath originate? a. Obturator Membrane b. Iliotibial tract fascia c. Transveralis fascia d. Extensor retinacula fascia e. Cribriform Fascia

C. Fibula

Which bone in the leg is not involved in the knee joint? A. Femur B. Tibia C. Fibula D. All of the above are involved in the knee joint

c) tibia, fibula, and talus

Which bones are involved in the ankle joint? a) tibia and talus b) tibia and calcaneous c) tibia, fibula, and talus c) tibia, fibular, and calcaneous

B. Superior gluteal artery

Which branch is the largest branch of the internal iliac artery and passes posteriorly between the lumbosacral trunk and the S1 nerve? A. Inferior gluteal artery B. Superior gluteal artery C. Internal prudendal artery D. Sciatic artery

c. Suprapatellar bursa

Which bursa is the commonly injured during craweling? a. Deep patellar bursa b. Semimembranous bursa c. Suprapatellar bursa d. Subcutaneous infrapatellar Bursa e. Prepatellar Bursa

B) Vastus Lateralis

Which component of the quadriceps is the largest? A) Vastus Medialis B) Vastus Lateralis C) Vastus Intermedius D) Adductor longus

b) Femoral Artery

Which does NOT create a border of the femoral triangle a) Lateral Sartorius b) Femoral Artery c) Superior Inguinal Ligament d) Medial Adductor Longus

c) Hip and thigh contusion

Which injury is only second to sprains in athletes, causes bleeding from ruptured vesels, often a sports injury, presents as a hematoma, is a result from a direct blow from an opponents knee. a) Dislocated hip b) Femur fracture c) Hip and thigh contusion d) Broken pelvis

d) Apply heat or massage to facilitate myositis ossificans

Which is NOT an appropriate treatment for an intermuscular hematoma of the thigh? a) Cold therapy for 10 minutes every 2 hours b) Compression wrap to stop internal bleeding and deter edema c) Elevate to aid fluid drainage from injury site d) Apply heat or massage to facilitate myositis ossificans

A. common fibular

Which is a major branch of the sciatic nerve? A common fibular B deep fibular C femoral D posterior tibial

c. sacrospinous ligament

Which ligament divides the greater sciatic foramen into two compartments? a. posterior sacro-iliac ligament b. sacrotuberous ligament c. sacrospinous ligament d. ischiofemoral ligament

a. Sacrotuberous ligament

Which ligament is continuous (inferiorly) to the posterior sacroiliac ligament? a. Sacrotuberous ligament b. Sacrospinous ligament c. Ischiofemoral ligament d. Iliolumbar ligament

e. Sacrotuberous ligament

Which ligament subdivides the foramina created by the sacrotuberous ligament into the greater and lesser sciatic foramina? a. Posterior sacrococcygeal ligament b. Iliolumbar ligament c. Posterior sacro-iliac ligamet d. Ischiofemoral ligament e. Sacrotuberous ligament

c) anterior and posterior cruciate ligaments

Which ligaments crisscross the knee within the joint capsule? a) fibular and tibial collateral ligaments b) oblique and arcuate popliteal ligaments c) anterior and posterior cruciate ligaments d) None of the ligaments of the knee crisscross

d) Lateral gastrocnemius

Which lower extremity muscle is NOT part of the anterior compartment of the leg and responsible for dorsiflexion/extension of the foot? a) Extensor Digitorum Longus b) Fibularis tertius c) Extensor hallucis Longus d) Lateral gastrocnemius

A. 1st metatarsophalangeal joint

Which metatarsophalangeal joint is the first and commonly affect joint by gout and osteoarthritis? A. 1st metatarsophalangeal joint B. 2nd metatarsophalangeal joint C. 3rd metatarsophalangeal joint D. 4th metatarsophalangeal joint E. 5th metatarsophalangeal joint

c. Flexor digitorum longus

Which muscle has the following: proximal attachment it medial posterior tibia, distal attachment is bases of distal phalanges of lateral 4 digits, innervation by the tibial nerve (S2, S3), and is tested by having the distal phalanges of the lateral 4 toes flexed against resistance? a. Flexor hallucis longus b. Tibialis posterior c. Flexor digitorum longus d. Fibularis longus muscle

c. fibularis brevis

Which muscle is NOT a muscle of the anterior compartment of the leg? a. tibialis anterior b. extensor digitorum longus c. fibularis brevis d. extensor hallucis longus

D . sartorius

Which muscle is NOT affected in a rupture of the calcaneal tendon? A . gastrocnemius B . soleus C . plantaris D . sartorius

a. Fibularis brevis

Which muscle is innervated by the superficial fibular nerve and its main action is to evert the foot and weakly plantarflex the ankle? a. Fibularis brevis b. Fibularis tertius c. Extensor halluces longus d. Extensor digitorum longus

b) Iliopsoas

Which muscle is the chief flexor of the thigh? a) Pectineus b) Iliopsoas c) Sartorius d) Quadriceps femoris

A. Quadriceps femoris

Which muscle is the greatest extensor of the leg? A. Quadriceps femoris B. Sartorius C. Vastus medialis D. Vastus lateralis

c. plantaris

Which muscle of the posterior compartment of the leg originates at the lateral subcondylar ridge of femur, inserts at the posterior calcaneal tendon, and is absent in 10% of people? a. gastrocnemius b. popliteus c. plantaris d. tibialis posterior

A. Piriformis

Which muscle originates on the pelvic surface of the sacrum, passes through the greater sciatic notch, and inserts on the greater trochanter of the femur? A. Piriformis B. Gluteus minimus C. Superior gemellus D. Obturator internus E. Quadratus femoris

a) Pectineus

Which muscle's main action is to adduct and flex the thigh as well as assist with medial rotation of the thigh? a) Pectineus b) Psoas major c) Psoas minor d) Iliacus

c) Bicep femoris short head

Which muscle's proximal attachment is not attached to the ischial tuberosity? a) Semitendinosus b) Semimembranosus c) Bicep femoris short head d) Bicep femoris long head

B. hamstrings

Which muscles are the main extensors of the hip used in normal walking? A. gluteus maximus B. hamstrings C. gluteus medius D. biceps femoris

c) saphenous nerve

Which nerve innervates the skin on the medial side of the leg and foot? a) lateral plantar nerve b) medial plantar nerve c) saphenous nerve d) genitofemoral

a. Tibial nerve

Which nerve supplies all the muscles in the posterior compartment of the leg? a. Tibial nerve b. Fibular nerve c. Superior gluteal nerve d. Clunial nerves

d. Digital Branches of plantar arch

Which of the following DOES NOT supply the talar neck with blood? a. Posterior Tibial Artery b. Anterior Tibial Artery c. Perforating Peroneal Artery (via the artery of tarsal sinus d. Digital Branches of plantar arch

e. All of the above

Which of the following are causes of swollen lymph nodes? a. Immune system disorders b. Anti-seizure meds c. Infection from bacteria or viruses d. Cancer e. All of the above

d. great saphenous vein

Which of the following is NOT a content of the popliteal fossa? a. popliteal arteries b. popliteal lymph nodes c. tibual nerve d. great saphenous vein

a. posterior tibiotalar part

Which of the following is NOT a part of the lateral ligament of the ankle? a. posterior tibiotalar part b. anterior talofibular ligament c. posterior talofibular ligament d. calcaneofibular ligament

a. head of fibula

Which of the following is NOT one of the three bony landmarks that forms a triangle for guidance of joint aspiration of the knee? a. head of fibula b. anterolateral tibial tubercle c. lateral epicondyle of femur d. apex of the patella

c) It supplies many structures in the gluteal region.

Which of the following is NOT true about the sciatic nerve? a) It is the largest nerve in the body b) It is a continuation of the main part of the sacral plexus. c) It supplies many structures in the gluteal region. d) It is actually two nerves (tibial nerve and common fibular nerve) bound together in the same connective tissue sheath.

a. Infrapatellar synovial fold

Which of the following is considered a landmark for orthoscopic surgeons? a. Infrapatellar synovial fold b. Suprapatellar synovial fold c. Internal synovial membrane d. Medial meniscus e. Lateral meniscus

b. Both are innervated by tibial division of sciatic nerve Explanation: Long head is innervated by tibial division of sciatic nerve. Short head is innervated by fibular division of sciatic nerve

Which of the following is false regarding the two heads of the biceps femoris muscle? a. Both share a common tendon b. Both are innervated by tibial division of sciatic nerve c. If one head is injured, the other can still operate d. All of these are true

a. Fibula

Which of the following is not involved in the knee joint? a. Fibula b. tibula c. anterior cruciate ligamanet d. medial collateral ligament

d. femoral nerve

Which of the following is not surrounded by the femoral sheath? a. femoral artery b. femoral vein c. femoral lymphatics (deep inguinal lymph nodes) d. femoral nerve

b) flexible flat feet Flexible flat feet lack a medial arch, when weight bearing but are normal in appearance when not bearing weight. Flexible flat feet is more common, and is a result from loose or degenerated intrinsic ligaments (inadequate passive arch support). This is common in childhood but usually resolves with age as the ligaments grow and mature. The condition occasionally persists into adulthood and may or may not be symptomatic.

Which of the following is the condition of flat feet, lacking a medial arch when weight bearing but normal in appearance when not bearing weight? a) rigid flat feet b) flexible flat feet c) acquired flat feet d) congenital flat feet

b) common fibular nerve

Which of the following nerves is most often injured in the lower limb? a) tibial nerve b) common fibular nerve c) superficial fibular nerve d) deep fibular nerve

d. pain out of proportion

Which of the following symptoms is indicative of compartment syndrome? a. peripheral edema b. pruritus c. pleuritic chest pain d. pain out of proportion

b) osteofibrous dysplsia

Which of the following would most likely be consistent with a "soap bubble appearance" on XR? a) osteogenesis imperfecta b) osteofibrous dysplsia c) osteoporosis d) trampoline fracture

a) Gluteus maximus

Which of the gluteal muscles extends the thigh? a) Gluteus maximus b) Gluteus medius c) Gluteus minimus d) Gluteus lateralus

D. Gracilis

Which thigh muscle in the medial compartment (adductors) of the thigh is the weakest? A. Adductor longus B. Adductor brevis C. Adductor magnus D. Gracilis E. Obturator externus

a. Greater saphenous vein

Which vein is commonly used for CABG due to its readily accessibility, sufficient distance between tributaries and perforating veins, and higher percentage of muscular and elastic fibers within the wall? a. Greater saphenous vein b. Lesser saphenous vein c. Popliteal vein d. Profunda femoris vein

c) great saphenous vein "The great saphenous vein is sometimes used for coronary arterial bypasses because (1) it is readily accessible, (2) a sufficient distance occurs between the tributaries and the perforating veins so that usable lengths can be harvested, and (3) its wall contains a higher percentage of muscular and elastic fibers than do other superficial veins."

Which vein is commonly used for coronary arterial bypasses? a) fibular vein b) popliteal vein c) great saphenous vein d) anterior tibial vein

B. Great saphenous vein

Which vein runs superficially along the medial aspect of the lower leg? A. Lesser saphenous vein B. Great saphenous vein C. Femoral vein D. Fibular vein

b) Inability to properly walk. Pain during activity and palpation. Movement against resistance is painful

Which would classify a grade II Thigh contusion/hematoma? a) Tightness in thigh, may or may not have swelling. Nearly full range of motion b) Inability to properly walk. Pain during activity and palpation. Movement against resistance is painful c) Inability to walk without crutches. Severe pain and significant swelling. d) Inability to walk. Severe pain and significant swelling.

D) Inferior to the inguinal ligament

While practicing for the cardiovascular exam for the H&P practical, you are asked to palpate and grade the femoral artery. Being an excellent PA student who is proficient at peripheral vasculature, you know this location can also find the femoral vein, which is medial to it. Where can the pulsations be felt? A) Posterior thigh compartment B) Medial to iliac crest of hip bone C) Behind the knee (popliteal) D) Inferior to the inguinal ligament

c) It is deep and protected by its position in the politeal fosssa

Why is injury to the tibial nerve uncommon? a) The tibia is such a strong bone b) People rarely hit their shin's c) It is deep and protected by its position in the politeal fosssa d) It has a protective sheath around it that is stronger than other protective sheaths.

a. so the valves of the vein do not obstruct blood flow

Why must the great saphenous vein be turned inside out before being grafted into the coronary arteries? a. so the valves of the vein do not obstruct blood flow b. because the outside of the vein is stronger and needed for the increased pressure of the coronaries c. in case there is previous damage on the inside on the vein d. the great saphenous vein does not need to be turned inside out

B . Medial side

With respect to the sciatic nerve, the buttocks have a side of safety and a side of danger. Which side is the "side of danger"? A . Lateral side B . Medial side C . Anterior side D . Posterior side

A. Sciatic nerve

You are a PA that needs to give a Penicillin IM injection in the clinic. IM injections are common in the gluteal region because the muscles are thick and large; providing substantial volume for absorption of injected substances. You recall that this area is relatively safe but remember that it can be dangerous if not injected into the correct spot because of what underlying structure? A. Sciatic nerve B. Femoral artery C. Inguinal lymph nodes D. Sacrum

C: The fibula, this does not impact the patient's ability to walk

You are an orthopedic surgeon and have just removed a large section of the humeral shaft of a patient with an osteosarcoma. To save the arm from amputation you elect to proceed with a bone graft. Where do you harvest the graft from? A: The femur, the femur is large and will be unaffected B: The tibia, this bone is the most similar to the humerus C: The fibula, this does not impact the patient's ability to walk D: The contralateral humerus, it is best to use the same bone from the other side of the body

b) positive Trendelenburg test

You are evaluating Maddie in the orthopedic clinic who you suspect of having an injury to her superior gluteal nerve. You ask her to stand one one leg. You notice as she does this her pelvis on the unsupported side descends indicated that her gluteus medius and minimus on the supported side are weak or nonfunctional. This sign is known as: a) negative Trendelenburg test b) positive Trendelenburg test c) negative Hardenburg d) positive Hardenburg

B. You must be careful to identify the great saphenous vein and not tie off the femoral by mistake

You are performing an operation to ablate varicose veins on a very thin patient. What must you be aware of. A. Nothing, proceed as usual B. You must be careful to identify the great saphenous vein and not tie off the femoral by mistake C. You must tie off the femoral vein D. It's okay if you tie off the femoral vein, there are other tributaries so blood will still flow

D femoral hernia

You are seeing a patient in the clinic. She is a mother to four children, and complains of a recent onset of hip and abdominal pain. In the physical exam you feel a bulge inferolateral to her pubic tubercle. What is high on your differential? A anneurysm B groin pull C laceration of femoral artery D femoral hernia

c. popliteal fossa

You have a patient come in with a purulent infection in her lateral compartment of her leg. What areas are you concerned the infection may spread to? a. anterior compartment b. posterior compartment c. popliteal fossa d. calcaneal bursa

a. hamstring injury

You have a patient come into the ER with intense pain and cannot put any pressure on his left leg. He tells you he was late to his basketball practice and wasn't able to warm up properly. Which of the following would be a logical diagnosis? a. hamstring injury b. ischial bursitis c. calcaneal tendinitis d. tibialis anterior strain

C: Psoas abscess

You have a patient present to you complaining of severe hip pain. While taking the patient's history you learn that the patient was recently diagnosed with tuberculosis of the vertebral column after a trip to Asia. What unusual pathology related to this new diagnosis may be causing this patient's hip pain? A: Osteoarthritis B: Rheumatoid arthritis C: Psoas abscess D: Hip fracture

b) tibialis posterior

Your next appointment of the day is a 28-year-old male presenting to the clinic for a routine physical. You've never seen this patient before, so you decide to quickly flip through his chart before he arrives to familiarize yourself with his medical background. You see that he has "acquired flat feet," otherwise known as fallen arches. Which muscle is associated with this phenomenon? a) tibialis anterior b) tibialis posterior c) flexor digitorum longus d) flexor digitorum brevis


Related study sets

CCNA 1 Version 5.1 Final Exam Answers 2016

View Set

NCLEX Prep: Client Needs: Psychosocial Integrity (I)

View Set

SECURITY + SPOOFING AND POISONING 5.2

View Set

FALL-ECO 120 SURVEY OF ECONOMICS

View Set

Yale ~ Science of Well-Being ~ Quiz 2

View Set